LE3 - REVIEWER Thyroid Flashcards

1
Q

Which of the following statements about thyroid physiology in pregnancy is correct?

A. Thyroid hormone is not essential for fetal brain development until after the fetal thyroid gland is functional.
B. Maternal serum thyroid-stimulating hormone (TSH) increases significantly in early pregnancy due to increased metabolic demands.
C. Human chorionic gonadotropin (hCG) can weakly stimulate TSH receptors, leading to lower TSH levels in early pregnancy.
D. Maternal thyroid hormone does not cross the placenta, so fetal thyroid development is independent of maternal thyroid function.

A

C. Human chorionic gonadotropin (hCG) can weakly stimulate TSH receptors, leading to lower TSH levels in early pregnancy.
Rationale: hCG structurally mimics TSH and can stimulate TSH receptors, causing a transient decrease in maternal TSH levels during early pregnancy. Maternal thyroid hormone is essential for fetal brain development, especially before the fetal thyroid becomes functional.

How well did you know this?
1
Not at all
2
3
4
5
Perfectly
2
Q

Which of the following changes occurs in maternal thyroid function during the first trimester of pregnancy?

A. Decreased production of thyroxine-binding globulin (TBG)
B. Increased secretion of thyrotropin-releasing hormone (TRH)
C. Decreased TSH levels due to hCG stimulation of TSH receptors
D. Decreased free thyroxine (T4) levels

A

C. Decreased TSH levels due to hCG stimulation of TSH receptors
Rationale: During early pregnancy, the increase in hCG causes a decrease in TSH levels due to weak stimulation of TSH receptors. Thyroxine-binding globulin (TBG) levels increase, and free T4 levels initially rise, leading to the suppression of TRH.

How well did you know this?
1
Not at all
2
3
4
5
Perfectly
3
Q

What percentage of fetal thyroxine (T4) is derived from maternal sources after the fetal thyroid gland begins functioning?

A. 10%
B. 20%
C. 30%
D. 50%

A

C. 30%
Rationale: Approximately 30% of fetal thyroxine (T4) comes from maternal sources even after the fetal thyroid begins synthesizing its own thyroid hormone. This maternal contribution is critical for normal brain development.

How well did you know this?
1
Not at all
2
3
4
5
Perfectly
4
Q

What is the primary reason for a decrease in maternal TSH levels in early pregnancy?

A. Increased production of thyrotropin-releasing hormone (TRH)
B. Increased thyroid hormone production by the fetus
C. Weak stimulation of TSH receptors by high levels of hCG
D. Direct suppression by progesterone

A

C. Weak stimulation of TSH receptors by high levels of hCG
Rationale: hCG, produced in large quantities during early pregnancy, has a structural similarity to TSH, leading to weak stimulation of the TSH receptor and resulting in lower maternal TSH levels.

How well did you know this?
1
Not at all
2
3
4
5
Perfectly
5
Q

A 28-year-old woman in her first trimester of pregnancy presents for a routine prenatal visit. She reports mild fatigue but denies any other symptoms. Her laboratory results show slightly elevated free T4 levels and a decreased TSH level. She is concerned about these findings and asks if they will affect her pregnancy.

Question: What is the most likely explanation for her lab results, and what advice should you provide her?

A. She has subclinical hyperthyroidism and should be started on antithyroid medications.
B. This is a normal physiological change in early pregnancy due to elevated hCG levels, and no treatment is necessary.
C. She has overt hypothyroidism, and levothyroxine should be initiated.
D. These results suggest Graves’ disease, and she should be referred to an endocrinologist for further evaluation.

A

B. This is a normal physiological change in early pregnancy due to elevated hCG levels, and no treatment is necessary.
Rationale: In early pregnancy, hCG can cause a mild increase in free T4 and suppression of TSH. This is a normal physiological response, and treatment is not required unless other symptoms or significant thyroid dysfunction is present.

How well did you know this?
1
Not at all
2
3
4
5
Perfectly
6
Q

A 32-year-old woman with a history of Graves’ disease that was treated with radioiodine therapy two years ago becomes pregnant. She is currently asymptomatic. During her first prenatal visit, her TSH is found to be low, and free T4 is slightly elevated.

Question: What is the most appropriate next step in managing her thyroid function during pregnancy?

A. Start her on a low dose of methimazole.
B. Continue to monitor her thyroid function closely without starting any treatment.
C. Initiate high-dose propylthiouracil (PTU) to control her thyroid hormone levels.
D. Administer iodine supplementation to reduce thyroid hormone production.

A

B. Continue to monitor her thyroid function closely without starting any treatment.
Rationale: Pregnancy often leads to a remission of Graves’ disease due to the immune modulation that occurs during this period. The mildly elevated free T4 and low TSH could be due to hCG-related TSH suppression rather than recurrent Graves’ disease, so close monitoring without immediate treatment is recommended unless overt hyperthyroidism develops.

How well did you know this?
1
Not at all
2
3
4
5
Perfectly
7
Q

A 25-year-old pregnant woman at 10 weeks of gestation presents with fatigue, weight gain, and cold intolerance. Her thyroid function tests reveal an elevated TSH level and low free T4.

Question: What is the potential risk to her fetus if her hypothyroidism is not properly treated during pregnancy?

A. Increased risk of fetal hyperthyroidism
B. Increased risk of fetal hypothyroidism and impaired brain development
C. Increased maternal risk of postpartum thyroiditis
D. Increased fetal risk of congenital heart disease

A

B. Increased risk of fetal hypothyroidism and impaired brain development
Rationale: Untreated maternal hypothyroidism can lead to inadequate thyroid hormone transfer to the fetus, which is critical for normal brain development, especially before the fetal thyroid begins functioning around 12 weeks of gestation.

How well did you know this?
1
Not at all
2
3
4
5
Perfectly
8
Q

Which of the following best describes the pathophysiology of Graves’ disease?

A. Thyroid-stimulating blocking antibodies prevent the thyroid from producing hormones.
B. Thyroid-stimulating autoantibodies attach to thyrotropin receptors, causing hyperthyroidism.
C. Thyroid peroxidase antibodies destroy thyroid tissue, leading to hypothyroidism.
D. Fetal lymphocytes attack the maternal thyroid, causing thyroid dysfunction.

A

B. Thyroid-stimulating autoantibodies attach to thyrotropin receptors, causing hyperthyroidism.
Rationale: In Graves’ disease, thyroid-stimulating autoantibodies bind to the TSH receptor and activate it, causing overproduction of thyroid hormones and thyroid gland enlargement.

How well did you know this?
1
Not at all
2
3
4
5
Perfectly
9
Q

Thyroid peroxidase antibodies (TPO-Ab) are associated with which of the following pregnancy complications?

A. Preeclampsia
B. Gestational diabetes
C. Placental abruption and increased risk of abortion
D. Macrosomia

A

C. Placental abruption and increased risk of abortion
Rationale: TPO antibodies are present in 10-20% of pregnant women and are associated with autoimmune thyroiditis, which can lead to thyroid failure, increased risk of abortion, and placental abruption.

How well did you know this?
1
Not at all
2
3
4
5
Perfectly
10
Q

Which mechanism explains why autoimmune thyroid disease is more common in females than males?

A. Women have higher estrogen levels, which stimulate thyroid antibodies.
B. Fetal lymphocytes persist in the maternal circulation, leading to immune dysregulation.
C. Male fetuses transfer autoimmune antibodies to the mother during pregnancy.
D. Females have a stronger immune system that is more prone to attack the thyroid.

A

B. Fetal lymphocytes persist in the maternal circulation, leading to immune dysregulation.
Rationale: Fetal-to-maternal cell trafficking allows fetal lymphocytes to persist in maternal circulation for up to 20 years, contributing to the development of autoimmune thyroid diseases in women.

How well did you know this?
1
Not at all
2
3
4
5
Perfectly
11
Q

Which autoimmune thyroid disease is potentially caused by the maternal microchimerism of male fetal cells expressing the SRY sex-determining gene?

A. Graves’ disease
B. Hashimoto’s thyroiditis
C. Subacute thyroiditis
D. Papillary thyroid carcinoma

A

B. Hashimoto’s thyroiditis
Rationale: Maternal microchimerism, where male fetal cells expressing the SRY gene remain in maternal circulation, has been linked to the development of Hashimoto’s thyroiditis, an autoimmune thyroid condition.

How well did you know this?
1
Not at all
2
3
4
5
Perfectly
12
Q

Case Scenario:
A 30-year-old pregnant woman in her first trimester presents with fatigue, weight loss, and palpitations. Her thyroid function tests show low TSH and elevated free T4. She also tests positive for thyroid-stimulating autoantibodies.

Question: What is the most likely diagnosis and the underlying mechanism of her condition?

A. Hashimoto’s thyroiditis; thyroid destruction by thyroid peroxidase antibodies.
B. Graves’ disease; activation of thyrotropin receptors by thyroid-stimulating autoantibodies.
C. Subacute thyroiditis; inflammation of the thyroid gland causing hormone release.
D. Euthyroid; physiological changes in pregnancy leading to temporary thyroid hormone imbalance.

A

B. Graves’ disease; activation of thyrotropin receptors by thyroid-stimulating autoantibodies.
Rationale: The patient presents with classic hyperthyroid symptoms (weight loss, palpitations, fatigue), and her lab results show low TSH and high T4. The presence of thyroid-stimulating autoantibodies indicates Graves’ disease, where these antibodies activate the TSH receptor, causing hyperthyroidism.

`

How well did you know this?
1
Not at all
2
3
4
5
Perfectly
13
Q

A 35-year-old woman with a history of Hashimoto’s thyroiditis presents to the clinic at 20 weeks of pregnancy. Her thyroid peroxidase antibody (TPO-Ab) test is positive. She is concerned about how this may affect her pregnancy.

Question: What is the most appropriate counseling for this patient regarding the risks associated with TPO antibodies in pregnancy?

A. TPO antibodies will likely cause hyperthyroidism, increasing the risk of preeclampsia.
B. TPO antibodies may lead to thyroid failure, increasing the risk of abortion and placental abruption.
C. TPO antibodies are benign and have no effect on the pregnancy outcome.
D. TPO antibodies will cause congenital hypothyroidism in the fetus.

A

B. TPO antibodies may lead to thyroid failure, increasing the risk of abortion and placental abruption.
Rationale: TPO antibodies are associated with autoimmune thyroiditis and thyroid failure. This increases the risk of pregnancy complications such as abortion and placental abruption, and the patient should be monitored closely for thyroid function throughout the pregnancy.

How well did you know this?
1
Not at all
2
3
4
5
Perfectly
14
Q

A 32-year-old woman with a history of three pregnancies, including one male child, presents with fatigue and weight gain 18 months postpartum. She reports a strong family history of autoimmune diseases. Laboratory testing reveals elevated TSH and positive TPO antibodies.

Question: What is the most likely diagnosis, and what mechanism might explain the development of her condition?

A. Graves’ disease; thyroid-stimulating antibodies cross-reacting with fetal antigens.
B. Subacute thyroiditis; inflammation from fetal microchimerism.
C. Hashimoto’s thyroiditis; fetal lymphocytes from her male child contributing to autoimmune thyroiditis.
D. Postpartum thyroiditis; normal fluctuation of thyroid hormones after pregnancy.

A

C. Hashimoto’s thyroiditis; fetal lymphocytes from her male child contributing to autoimmune thyroiditis.
Rationale: The patient’s elevated TSH and positive TPO antibodies suggest Hashimoto’s thyroiditis, an autoimmune thyroid disease. The mechanism may involve maternal microchimerism, where fetal lymphocytes from her male child persist in her circulation and contribute to the autoimmune process.

How well did you know this?
1
Not at all
2
3
4
5
Perfectly
15
Q

What is the most common symptom of hyperthyroidism in pregnancy?

A. Weight gain despite increased appetite
B. Bradycardia
C. Tachycardia with elevated sleeping pulse rate
D. Hypotension

A

C. Tachycardia with elevated sleeping pulse rate
Rationale: Tachycardia, especially during rest (elevated sleeping pulse rate), is a hallmark sign of hyperthyroidism in pregnancy due to the overproduction of thyroid hormones.

How well did you know this?
1
Not at all
2
3
4
5
Perfectly
16
Q

Which thioamide drug is preferred for treating hyperthyroidism during the first trimester of pregnancy?

A. Methimazole
B. Propylthiouracil (PTU)
C. Levothyroxine
D. Radioactive iodine

A

B. Propylthiouracil (PTU)
Rationale: PTU is the preferred treatment for hyperthyroidism in the first trimester due to a lower risk of teratogenic effects compared to methimazole, which is associated with birth defects when used early in pregnancy.

How well did you know this?
1
Not at all
2
3
4
5
Perfectly
17
Q

Which of the following is a known adverse effect of propylthiouracil (PTU) that warrants discontinuation of the drug?

A. Transient leukopenia
B. Agranulocytosis
C. Mild rash
D. Weight gain

A

B. Agranulocytosis
Rationale: Agranulocytosis, a severe decrease in white blood cell count, is a rare but serious side effect of PTU. The drug should be discontinued immediately if the patient develops symptoms such as fever or sore throat, which may indicate this condition.

How well did you know this?
1
Not at all
2
3
4
5
Perfectly
18
Q

Why is methimazole avoided during the first trimester of pregnancy?

A. It causes hepatotoxicity in the mother.
B. It is associated with fetal congenital defects such as esophageal atresia and aplasia cutis.
C. It is ineffective at controlling hyperthyroidism.
D. It causes maternal hypothyroidism.

A

B. It is associated with fetal congenital defects such as esophageal atresia and aplasia cutis.
Rationale: Methimazole is associated with specific congenital malformations, including esophageal atresia and aplasia cutis, when used in the first trimester. Therefore, it is avoided during early pregnancy and preferred in the second trimester.

How well did you know this?
1
Not at all
2
3
4
5
Perfectly
19
Q

Which of the following treatments for hyperthyroidism is contraindicated in pregnancy?

A. Propylthiouracil (PTU)
B. Methimazole
C. Subtotal thyroidectomy
D. Radioactive iodine ablation

A

D. Radioactive iodine ablation
Rationale: Radioactive iodine ablation is contraindicated during pregnancy because it can destroy the fetal thyroid gland and result in fetal hypothyroidism or abortion.

How well did you know this?
1
Not at all
2
3
4
5
Perfectly
20
Q

Case Scenario:
A 26-year-old woman, who is 10 weeks pregnant, presents with palpitations, weight loss despite adequate food intake, and tremors. Her physical exam reveals tachycardia, a mildly enlarged thyroid gland, and mild exophthalmos. Laboratory results show elevated free T4 and suppressed TSH levels.

Question: What is the most appropriate initial treatment for this patient?

A. Methimazole 10-40 mg/day
B. Radioactive iodine therapy
C. Propylthiouracil (PTU) 50-150 mg 3 times daily
D. Subtotal thyroidectomy

A

C. Propylthiouracil (PTU) 50-150 mg 3 times daily
Rationale: PTU is the preferred treatment for hyperthyroidism in the first trimester of pregnancy due to its lower risk of causing fetal malformations. Methimazole should be avoided during this period due to its association with congenital defects.

1=0-13
2=14-27
3=28-37

How well did you know this?
1
Not at all
2
3
4
5
Perfectly
21
Q

A 32-year-old pregnant woman in her second trimester is being treated for hyperthyroidism with propylthiouracil (PTU). During her follow-up visit, she reports a sore throat and fever. Her lab work shows a low white blood cell count.

Question: What is the next best step in management?

A. Continue PTU and monitor her symptoms.
B. Discontinue PTU and initiate methimazole.
C. Refer the patient for radioactive iodine therapy.
D. Increase the PTU dose and repeat blood work in 2 weeks.

A

B. Discontinue PTU and initiate methimazole.
Rationale: The patient’s symptoms and low white blood cell count suggest agranulocytosis, a known side effect of PTU. The drug should be discontinued, and methimazole, which is safer in the second trimester, should be started.

How well did you know this?
1
Not at all
2
3
4
5
Perfectly
22
Q

A 30-year-old woman who is 25 weeks pregnant is diagnosed with uncontrolled hyperthyroidism. Despite medical therapy with PTU, her symptoms persist. After discussing treatment options, the decision is made to proceed with surgery.

Question: What is the most appropriate timing for performing a subtotal thyroidectomy in this patient?

A. First trimester
B. Second trimester
C. Third trimester
D. Immediately postpartum

A

B. Second trimester
Rationale: If surgery is required for hyperthyroidism during pregnancy, the second trimester is the safest time to perform a thyroidectomy because it minimizes the risks to both the mother and fetus. Surgery is generally avoided during the first and third trimesters due to the higher risks of miscarriage and preterm labor.

How well did you know this?
1
Not at all
2
3
4
5
Perfectly
23
Q

A 28-year-old pregnant woman in her third trimester presents with a history of hyperthyroidism managed with methimazole. At a routine prenatal visit, her physician expresses concern that the baby may have been exposed to the drug.

Question: What complication should be considered in the newborn due to methimazole exposure?

A. Fetal hypothyroidism
B. Congenital aplasia cutis
C. Hyperthyroidism
D. Fetal thyroid gland destruction

A

B. Congenital aplasia cutis
Rationale: Methimazole use during the first trimester is associated with congenital malformations, including aplasia cutis, a condition characterized by the absence of a portion of the skin. This is why methimazole is avoided in the first trimester, although it is considered safe in later stages of pregnancy.

How well did you know this?
1
Not at all
2
3
4
5
Perfectly
24
Q

What is the characteristic laboratory finding in subclinical hyperthyroidism?

A. Elevated TSH with normal T4 and T3 levels
B. Low TSH with normal T4 and T3 levels
C. Elevated TSH with elevated T4 levels
D. Low TSH with elevated T4 levels

A

B. Low TSH with normal T4 and T3 levels
Rationale: Subclinical hyperthyroidism is defined by a low TSH level with normal thyroid hormone (T4 and T3) levels. This differentiates it from overt hyperthyroidism, where T4 and/or T3 levels would be elevated.

How well did you know this?
1
Not at all
2
3
4
5
Perfectly
25
Q

Which of the following complications is associated with subclinical hyperthyroidism?

A. Bradycardia
B. Osteopenia
C. Hypothyroidism
D. Decreased bone turnover

A

B. Osteopenia
Rationale: Subclinical hyperthyroidism can lead to decreased bone density (osteopenia), increasing the risk of fractures due to the effects of mild thyroid hormone excess on bone metabolism.

How well did you know this?
1
Not at all
2
3
4
5
Perfectly
26
Q

What is the most appropriate management for a patient diagnosed with subclinical hyperthyroidism and no significant symptoms?

A. Start antithyroid medications
B. Radioactive iodine ablation
C. No treatment is required
D. Thyroidectomy

A

C. No treatment is required
Rationale: In most cases of subclinical hyperthyroidism, especially if asymptomatic, no treatment is necessary. However, the condition should be monitored for progression or complications.

How well did you know this?
1
Not at all
2
3
4
5
Perfectly
27
Q

Which of the following cardiovascular conditions can occur due to subclinical hyperthyroidism?

A. Ventricular hypertrophy
B. Myocardial infarction
C. Pericarditis
D. Atrial stenosis

A

A. Ventricular hypertrophy
Rationale: Subclinical hyperthyroidism can cause cardiovascular changes, including ventricular hypertrophy, due to increased heart activity related to thyroid hormone effects on the cardiovascular system.

How well did you know this?
1
Not at all
2
3
4
5
Perfectly
28
Q

Which of the following is NOT a known potential consequence of subclinical hyperthyroidism?

A. Cardiac arrhythmias
B. Osteopenia
C. Ventricular hypertrophy
D. Adrenal insufficiency

A

D. Adrenal insufficiency
Rationale: Subclinical hyperthyroidism is not associated with adrenal insufficiency. It is primarily linked to cardiac complications (arrhythmias, ventricular hypertrophy) and bone density reduction (osteopenia).

How well did you know this?
1
Not at all
2
3
4
5
Perfectly
29
Q

Case Scenario:
A 65-year-old woman with no significant medical history is found to have a serum TSH level of 0.2 mU/L (reference range: 0.4-4.0 mU/L) during routine blood work. Her free T4 and T3 levels are within normal limits. She reports feeling well with no symptoms of hyperthyroidism.

Question: What is the most appropriate next step in managing this patient?

A. Start her on methimazole.
B. Perform a radioactive iodine uptake scan.
C. No treatment is required; monitor thyroid function periodically.
D. Refer for thyroidectomy.

A

C. No treatment is required; monitor thyroid function periodically.
Rationale: This patient has subclinical hyperthyroidism, characterized by low TSH with normal T4 and T3 levels. Since she is asymptomatic, treatment is not needed, but her thyroid function should be monitored for any changes or progression to overt hyperthyroidism.

How well did you know this?
1
Not at all
2
3
4
5
Perfectly
30
Q

A 70-year-old man with a history of atrial fibrillation presents for a follow-up visit. Recent laboratory tests show a low TSH level of 0.1 mU/L, with normal T4 and T3 levels. He is concerned about the potential risks of his thyroid condition.

Question: What complication is most likely associated with his subclinical hyperthyroidism and should be closely monitored?

A. Bradycardia
B. Osteoporosis
C. Progression to hypothyroidism
D. Cardiac arrhythmias

A

D. Cardiac arrhythmias
Rationale: Subclinical hyperthyroidism is associated with an increased risk of cardiac arrhythmias, particularly in older adults with pre-existing heart conditions like atrial fibrillation. Monitoring and managing cardiac health is crucial in these patients.

How well did you know this?
1
Not at all
2
3
4
5
Perfectly
31
Q

A 60-year-old woman with subclinical hyperthyroidism presents with a bone density scan showing signs of osteopenia. She is concerned about her thyroid condition contributing to bone loss.

Question: What is the best advice regarding the management of her subclinical hyperthyroidism in light of her osteopenia?

A. Start treatment with antithyroid medication to prevent further bone loss.
B. Increase calcium and vitamin D intake, but no specific treatment for thyroid is needed.
C. Schedule thyroid surgery to prevent complications.
D. Start her on radioactive iodine therapy.

A

B. Increase calcium and vitamin D intake, but no specific treatment for thyroid is needed.
Rationale: Subclinical hyperthyroidism can contribute to bone loss, but antithyroid treatment is not typically recommended unless there is significant progression. Calcium and vitamin D supplementation can help manage the osteopenia while thyroid function is monitored.

How well did you know this?
1
Not at all
2
3
4
5
Perfectly
32
Q

A 55-year-old woman presents with palpitations and a recent diagnosis of subclinical hyperthyroidism. Her TSH is 0.3 mU/L, and her T4 and T3 levels are normal. She is asymptomatic aside from occasional palpitations.

Question: What is the most appropriate management plan for this patient at this stage?

A. Start beta-blockers for symptomatic management of palpitations.
B. Prescribe methimazole to normalize TSH levels.
C. Refer her for radioactive iodine ablation therapy.
D. Perform a subtotal thyroidectomy.

A

A. Start beta-blockers for symptomatic management of palpitations.
Rationale: Beta-blockers can be used to manage palpitations in patients with subclinical hyperthyroidism, especially if symptoms like tachycardia or arrhythmias occur. Antithyroid drugs or more invasive treatments are not indicated unless progression to overt hyperthyroidism occurs.

How well did you know this?
1
Not at all
2
3
4
5
Perfectly
33
Q

Which of the following is a common precipitating factor for thyroid storm in pregnant women?

A. Gestational diabetes
B. Preeclampsia
C. Hyperemesis gravidarum
D. Iron deficiency

A

B. Preeclampsia
Rationale: Preeclampsia is a well-known precipitating factor for thyroid storm in pregnant women, especially due to its stress on cardiovascular and metabolic systems.

Precipitating Factors:
Preeclampsia, anemia, and sepsis are common triggers in pregnant women with thyrotoxicosis.

How well did you know this?
1
Not at all
2
3
4
5
Perfectly
34
Q

Which of the following is the most appropriate first-line treatment to block thyroid hormone synthesis during a thyroid storm?

A. Methimazole
B. Propylthiouracil (PTU)
C. Radioactive iodine
D. Lugol solution

A

B. Propylthiouracil (PTU)
Rationale: PTU is preferred in thyroid storm as it not only blocks thyroid hormone synthesis but also inhibits the peripheral conversion of T4 to the more active T3 form.

How well did you know this?
1
Not at all
2
3
4
5
Perfectly
35
Q

In the management of thyroid storm, iodine should be administered after what duration following PTU therapy?

A. 1-2 hours
B. 12-24 hours
C. 24-48 hours
D. Immediately after PTU

A

A. 1-2 hours
Rationale: Iodine should be administered 1-2 hours after PTU to inhibit thyroid hormone release. Administering it earlier can fuel the thyroid hormone production if hormone synthesis is not already blocked

How well did you know this?
1
Not at all
2
3
4
5
Perfectly
36
Q

What is the primary goal of corticosteroid therapy in thyroid storm?

A. Block the release of thyroid hormone from the thyroid gland
B. Block peripheral conversion of T4 to T3
C. Promote renal clearance of excess thyroid hormone
D. Stimulate the synthesis of thyroid hormone

A

B. Block peripheral conversion of T4 to T3
Rationale: Corticosteroids, such as dexamethasone, are used in thyroid storm to block the peripheral conversion of T4 to T3, which helps reduce the active thyroid hormone load in the body.

How well did you know this?
1
Not at all
2
3
4
5
Perfectly
37
Q

A 30-year-old woman who is 33 weeks pregnant presents with severe agitation, tachycardia, sweating, and fever. She has a known history of untreated Graves’ disease. Her pulse is 140 bpm, and her blood pressure is 140/90 mmHg. Laboratory tests show undetectable TSH and elevated free T4. The diagnosis of thyroid storm is made.

Question: What is the first-line therapy for this patient?

A. Methimazole and propranolol
B. Propylthiouracil (PTU) and propranolol
C. Iodine therapy and esmolol
D. Radioactive iodine

A

B. Propylthiouracil (PTU) and propranolol
Rationale: PTU is the first-line treatment in thyroid storm during pregnancy, as it blocks the synthesis of thyroid hormones and inhibits T4 to T3 conversion. Propranolol is used for heart rate control.

How well did you know this?
1
Not at all
2
3
4
5
Perfectly
38
Q

A 35-year-old woman presents at 36 weeks gestation with a history of Graves’ disease. She has developed heart failure and is diagnosed with thyroid storm. In addition to PTU and beta-blocker therapy, iodine therapy is considered. However, the patient has a known history of anaphylaxis to iodine-containing compounds.

Question: What is the best alternative therapy for this patient to control thyroid hormone release?

A. Administer Lugol’s solution
B. Administer lithium carbonate
C. Perform an emergency thyroidectomy
D. Administer dexamethasone to block hormone release

A

B. Administer lithium carbonate
Rationale: In cases of iodine anaphylaxis, lithium carbonate is used as an alternative to iodine therapy because it inhibits thyroid hormone release without triggering an allergic reaction.

How well did you know this?
1
Not at all
2
3
4
5
Perfectly
39
Q

A 33-year-old pregnant woman is admitted to the ICU for management of thyroid storm. Despite appropriate medical therapy with PTU, iodine, corticosteroids, and beta-blockers, she continues to have tachycardia and signs of decompensated heart failure. Echocardiography reveals dilated cardiomyopathy.

Question: What is the prognosis for her heart failure once her thyroid storm is adequately treated?

A. Permanent heart failure requiring lifelong management
B. Reversible heart failure if thyroid hormone levels are controlled
C. Requires immediate heart transplant
D. Heart failure will worsen regardless of thyroid treatment

A

B. Reversible heart failure if thyroid hormone levels are controlled
Rationale: Thyrotoxicosis-induced cardiomyopathy is reversible once thyroid hormone levels are controlled. The heart’s function can recover fully after stabilization of the hyperthyroid state.

How well did you know this?
1
Not at all
2
3
4
5
Perfectly
40
Q

A 40-year-old woman presents at 30 weeks of gestation with severe symptoms of thyroid storm. After stabilization, she asks about the risks to her pregnancy.

Question: What are the potential complications for her pregnancy if thyroid storm remains uncontrolled?

A. Gestational diabetes
B. Preterm delivery and heart failure
C. Polyhydramnios
D. Post-term delivery and macrosomia

A

B. Preterm delivery and heart failure
Rationale: Uncontrolled thyroid storm in pregnancy can lead to serious complications, including preterm delivery, preeclampsia, heart failure, and perinatal mortality.

How well did you know this?
1
Not at all
2
3
4
5
Perfectly
41
Q

Which beta-blocker is commonly used to control heart rate in patients with thyroid storm?

A. Atenolol
B. Metoprolol
C. Esmolol
D. Propranolol

A

D. Propranolol
Rationale: Propranolol is the beta-blocker of choice for controlling tachycardia in thyroid storm as it not only reduces heart rate but also inhibits peripheral conversion of T4 to T3.

How well did you know this?
1
Not at all
2
3
4
5
Perfectly
42
Q

Which drug should be used in thyroid storm patients with a history of iodine anaphylaxis?

A. Methimazole
B. Sodium iodide
C. Lugol solution
D. Lithium carbonate

A

D. Lithium carbonate
Rationale: Lithium carbonate is used as an alternative to iodine in patients with iodine anaphylaxis because it inhibits thyroid hormone release without triggering an allergic reaction.

How well did you know this?
1
Not at all
2
3
4
5
Perfectly
43
Q

A 29-year-old pregnant woman at 32 weeks gestation presents with thyroid storm and is treated with PTU and beta-blockers. After 1 hour, the team initiates iodine therapy. The patient improves, but her heart rate remains elevated at 130 bpm despite beta-blocker therapy. The team decides to switch to esmolol for better heart rate control.

Question: Why is esmolol preferred in this situation?

A. It has a longer duration of action.
B. It can be administered orally.
C. It has a rapid onset and short half-life, allowing for easy titration.
D. It increases the effectiveness of iodine therapy.

A

C. It has a rapid onset and short half-life, allowing for easy titration.
Rationale: Esmolol is ideal in acute settings like thyroid storm due to its rapid onset and short duration of action, allowing for precise control of heart rate.

How well did you know this?
1
Not at all
2
3
4
5
Perfectly
44
Q

A 32-year-old woman presents with tachycardia, fever, and altered mental status. She is 36 weeks pregnant and is diagnosed with thyroid storm. After stabilizing her with PTU, iodine, and beta-blockers, the team considers corticosteroid therapy.

Question: What is the rationale for using corticosteroids in thyroid storm management?

A. Reduce fever
B. Block peripheral conversion of T4 to T3
C. Promote excretion of thyroid hormones
D. Stabilize blood pressure

A

B. Block peripheral conversion of T4 to T3
Rationale: Corticosteroids like dexamethasone are used in thyroid storm to inhibit the peripheral conversion of T4 to T3, reducing the active hormone levels in circulation.

How well did you know this?
1
Not at all
2
3
4
5
Perfectly
45
Q

A 33-year-old pregnant woman with a history of hyperthyroidism presents to the emergency department in thyroid storm. After appropriate therapy with PTU, iodine, and beta-blockers, her condition stabilizes. The medical team manages her underlying precipitating condition, preeclampsia.

Question: Why is it important to control precipitating factors such as preeclampsia in thyroid storm?

A. To reduce the risk of anemia
B. To prevent worsening of thyroid hormone production
C. To improve cardiovascular stability
D. To enhance the effect of iodine therapy

A

C. To improve cardiovascular stability
Rationale: Managing precipitating factors like preeclampsia is crucial for cardiovascular stability, as pregnancy complications add strain to an already taxed cardiovascular system during thyroid storm.

How well did you know this?
1
Not at all
2
3
4
5
Perfectly
46
Q

What is the most common outcome for neonates born to mothers with Graves’ disease?

A. Hypothyroidism
B. Hyperthyroidism
C. Euthyroidism
D. Goiter with hyperthyroidism

A

C. Euthyroidism
Rationale: In most cases, neonates born to mothers with Graves’ disease are euthyroid (normal thyroid function), though around 1% may develop clinical hyperthyroidism.

How well did you know this?
1
Not at all
2
3
4
5
Perfectly
47
Q

What is the best predictor of perinatal thyrotoxicosis in a fetus born to a mother with Graves’ disease?

A. Maternal TSH levels
B. Fetal thyromegaly on ultrasound
C. Presence of thyroid-stimulating TSH-receptor antibodies in the mother
D. Maternal free T4 levels

A

C. Presence of thyroid-stimulating TSH-receptor antibodies in the mother
Rationale: The best predictor of perinatal thyrotoxicosis is the presence of thyroid-stimulating TSH-receptor antibodies in a mother with Graves’ disease. These antibodies can cross the placenta and affect the fetus.

How well did you know this?
1
Not at all
2
3
4
5
Perfectly
48
Q

Which fetal condition can result from maternal thioamide therapy during pregnancy?

A. Goitrous hypothyroidism
B. Fetal thyrotoxicosis
C. Non-goitrous hyperthyroidism
D. Fetal euthyroidism

A

A. Goitrous hypothyroidism
Rationale: Goitrous hypothyroidism can occur due to maternal administration of thioamides, which can reduce fetal thyroid hormone production, causing goiter and hypothyroidism in the fetus.

How well did you know this?
1
Not at all
2
3
4
5
Perfectly
49
Q

Which treatment is recommended for a fetus diagnosed with hypothyroidism and thyromegaly due to maternal Graves’ disease?

A. Maternal thioamides
B. Radioiodine 131I therapy
C. Intra-amniotic thyroxine
D. Fetal thyroidectomy

A

C. Intra-amniotic thyroxine
Rationale: For a hypothyroid fetus with thyromegaly, the recommended treatment is intra-amniotic thyroxine, which helps to correct the hypothyroid state.

How well did you know this?
1
Not at all
2
3
4
5
Perfectly
50
Q

What is the most likely cause of non-goitrous hypothyroidism in a fetus born to a mother with thyroid disease?

A. Transplacental passage of thyrotropin receptor-blocking antibodies
B. Maternal thioamide overdose
C. Fetal thyroid agenesis
D. Excess maternal TSH secretion

A

A. Transplacental passage of thyrotropin receptor-blocking antibodies
Rationale: Non-goitrous hypothyroidism in fetuses can result from the transplacental passage of thyrotropin receptor-blocking antibodies, which impair thyroid function without causing goiter.

How well did you know this?
1
Not at all
2
3
4
5
Perfectly
51
Q

A 34-year-old pregnant woman at 30 weeks gestation is being monitored for Graves’ disease. Recent blood tests reveal elevated maternal thyroid-stimulating TSH-receptor antibodies. Ultrasound shows fetal thyromegaly, and the mother is on thioamide therapy.

Question: What is the best next step to manage the fetus’s thyroid condition?

A. Continue maternal thioamide therapy
B. Perform intra-amniotic thyroxine injection
C. Initiate fetal blood sampling
D. Deliver the baby early

A

B. Perform intra-amniotic thyroxine injection
Rationale: The presence of fetal thyromegaly indicates possible hypothyroidism due to maternal thioamide therapy. The next step is to administer intra-amniotic thyroxine to correct fetal hypothyroidism.

How well did you know this?
1
Not at all
2
3
4
5
Perfectly
52
Q

A 28-year-old woman with a history of Graves’ disease was treated with radioiodine (131I) before pregnancy. At 32 weeks gestation, she undergoes ultrasound that shows normal fetal growth, but the fetus is found to have tachycardia. There is concern for fetal thyrotoxicosis.

Question: What is the likely cause of fetal thyrotoxicosis in this case?

A. Placental transfer of maternal thyrotropin receptor-blocking antibodies
B. Placental transfer of thyroid-stimulating antibodies
C. Overproduction of thyroid hormones by the fetal thyroid gland
D. Fetal exposure to maternal thioamides

A

B. Placental transfer of thyroid-stimulating antibodies
Rationale: Fetal thyrotoxicosis in this case is likely due to the transplacental passage of thyroid-stimulating antibodies from the mother. These antibodies stimulate the fetal thyroid gland, leading to increased hormone production.

How well did you know this?
1
Not at all
2
3
4
5
Perfectly
53
Q

A neonate is delivered at term to a mother with a 3-year history of Graves’ disease. The mother was treated with methimazole during pregnancy and was euthyroid at delivery. The infant, however, is diagnosed with hypothyroidism shortly after birth.

Question: What is the most likely cause of neonatal hypothyroidism in this case?

A. Fetal thyroid agenesis
B. Placental transfer of maternal thioamide
C. Neonatal thyroid-stimulating antibodies
D. Excessive iodine exposure during pregnancy

A

B. Placental transfer of maternal thioamide
Rationale: The neonate’s hypothyroidism is most likely due to placental transfer of methimazole (thioamide) from the mother. Thioamides can cross the placenta and inhibit thyroid hormone synthesis in the fetus, leading to hypothyroidism.

How well did you know this?
1
Not at all
2
3
4
5
Perfectly
54
Q

A 32-year-old pregnant woman with Graves’ disease is treated with PTU during her pregnancy. An ultrasound performed at 28 weeks shows fetal thyromegaly. The medical team is concerned about the fetal thyroid status and plans cord blood sampling.

Question: What is the purpose of performing cord blood sampling in this scenario?

A. To assess fetal adrenal function
B. To measure fetal thyroid hormone levels
C. To confirm maternal Graves’ disease
D. To evaluate fetal lung maturity

A

B. To measure fetal thyroid hormone levels
Rationale: Cord blood sampling is performed to measure fetal thyroid hormone levels and assess thyroid function in utero, particularly in cases where the fetus may be hypothyroid or thyrotoxic.

How well did you know this?
1
Not at all
2
3
4
5
Perfectly
55
Q

A 29-year-old woman with a history of Graves’ disease gives birth to a neonate with a goiter and signs of hypothyroidism. She had been treated with PTU throughout her pregnancy.

Question: What is the most appropriate treatment for this neonate?

A. Maternal PTU should be increased
B. Thyroid-stimulating hormone injections
C. Neonatal levothyroxine therapy
D. Neonatal corticosteroid therapy

A

C. Neonatal levothyroxine therapy
Rationale: The neonate with hypothyroidism and goiter likely needs levothyroxine therapy to restore normal thyroid function, especially since maternal PTU therapy has likely suppressed thyroid hormone production.

How well did you know this?
1
Not at all
2
3
4
5
Perfectly
56
Q

Which of the following is the primary cause of hyperthyroxinemia in hyperemesis gravidarum?

A. Increased secretion of TSH
B. Elevated levels of hCG
C. Decreased iodine intake
D. Increased production of thyroid antibodies

A

B. Elevated levels of hCG
Rationale: Hyperemesis gravidarum is associated with high levels of hCG, which can stimulate the thyroid gland, leading to increased thyroxine (T4) production and decreased TSH levels.

How well did you know this?
1
Not at all
2
3
4
5
Perfectly
57
Q

What is the recommended treatment for gestational thyrotoxicosis in patients with hyperemesis gravidarum?

A. Propylthiouracil (PTU)
B. Methimazole
C. No treatment
D. Radioactive iodine

A

C. No treatment
Rationale: Gestational thyrotoxicosis seen in hyperemesis gravidarum is transient and typically resolves by mid-pregnancy without the need for antithyroid medications.

How well did you know this?
1
Not at all
2
3
4
5
Perfectly
58
Q

Which of the following conditions is most commonly associated with high levels of hCG stimulating the TSH receptor?

A. Thyroiditis
B. Gestational trophoblastic disease
C. Postpartum thyroiditis
D. Hashimoto’s thyroiditis

A

B. Gestational trophoblastic disease
Rationale: Gestational trophoblastic disease (GTD) leads to markedly elevated hCG levels, which overstimulate the TSH receptor, causing increased thyroid hormone production and hyperthyroidism.

How well did you know this?
1
Not at all
2
3
4
5
Perfectly
59
Q

How do thyroid hormone levels typically change in hyperemesis gravidarum by mid-pregnancy?

A. They remain elevated throughout the pregnancy.
B. They normalize by mid-pregnancy.
C. They decrease further by mid-pregnancy.
D. They fluctuate throughout pregnancy.

A

B. They normalize by mid-pregnancy
Rationale: In hyperemesis gravidarum, thyroid hormone levels (T4) typically normalize by mid-pregnancy as hCG levels decline.

How well did you know this?
1
Not at all
2
3
4
5
Perfectly
60
Q

Which of the following is the main thyroid-related concern in gestational trophoblastic disease?

A. Hyperthyroidism due to high levels of hCG
B. Hypothyroidism due to decreased TSH secretion
C. Euthyroid state with no thyroid involvement
D. Goitrous hypothyroidism due to iodine deficiency

A

A. Hyperthyroidism due to high levels of hCG
Rationale: In gestational trophoblastic disease, the excessive hCG levels can overstimulate the TSH receptor, leading to hyperthyroidism.

How well did you know this?
1
Not at all
2
3
4
5
Perfectly
61
Q

A 26-year-old woman at 10 weeks of gestation presents with persistent nausea, vomiting, and weight loss. Blood tests show high free thyroxine (T4) and suppressed TSH levels. Her hCG levels are significantly elevated.

Question: What is the most likely diagnosis, and what is the appropriate management for her thyroid condition?

A. Graves’ disease; treat with methimazole.
B. Gestational thyrotoxicosis from hyperemesis gravidarum; no treatment needed.
C. Subclinical hyperthyroidism; start propylthiouracil.
D. Postpartum thyroiditis; monitor and initiate beta-blockers.

A

B. Gestational thyrotoxicosis from hyperemesis gravidarum; no treatment needed
Rationale: The patient’s symptoms, elevated T4, low TSH, and high hCG levels are consistent with gestational thyrotoxicosis secondary to hyperemesis gravidarum. Thyroid function typically normalizes without treatment by mid-pregnancy.

How well did you know this?
1
Not at all
2
3
4
5
Perfectly
62
Q

A 30-year-old woman with hyperemesis gravidarum at 12 weeks gestation is found to have high free T4 and undetectable TSH. Ultrasound shows a molar pregnancy, and her hCG levels are significantly elevated.

Question: What is the underlying mechanism for her thyroid dysfunction, and what condition is likely contributing to it?

A. Overstimulation of the thyroid by TSH; Hashimoto’s thyroiditis.
B. Overstimulation of the thyroid by hCG; gestational trophoblastic disease.
C. Direct stimulation of thyroid antibodies; postpartum thyroiditis.
D. Iodine deficiency; hypothyroidism.

A

B. Overstimulation of the thyroid by hCG; gestational trophoblastic disease
Rationale: In this case, the significantly elevated hCG due to a molar pregnancy (a form of gestational trophoblastic disease) is overstimulating the thyroid via the TSH receptor, causing hyperthyroidism.

How well did you know this?
1
Not at all
2
3
4
5
Perfectly
63
Q

A 29-year-old pregnant woman presents at 8 weeks gestation with severe vomiting, dehydration, and weight loss. Lab results show elevated free T4 and low TSH. Her physician diagnoses gestational thyrotoxicosis secondary to hyperemesis gravidarum.

Question: What is the expected course of her thyroid hormone levels, and when should they be monitored again?

A. Levels will normalize by mid-pregnancy; monitor in the second trimester.
B. Levels will remain elevated throughout pregnancy; start methimazole.
C. Levels will fluctuate throughout pregnancy; frequent monitoring needed.
D. Levels will drop below normal by mid-pregnancy; start levothyroxine.

A

A. Levels will normalize by mid-pregnancy; monitor in the second trimester
Rationale: In gestational thyrotoxicosis, thyroid hormone levels typically normalize by mid-pregnancy as hCG levels decrease, so no treatment is needed, but thyroid function should be rechecked in the second trimester to confirm normalization.

How well did you know this?
1
Not at all
2
3
4
5
Perfectly
64
Q

A 28-year-old woman is diagnosed with gestational trophoblastic disease at 10 weeks gestation, and blood tests show very high hCG levels along with symptoms of hyperthyroidism. Her doctor is concerned about the risk of hyperthyroidism worsening.

Question: What is the primary cause of her thyroid dysfunction, and what is the management approach?

A. Thyroid antibody overproduction; start corticosteroids.
B. TSH overproduction; monitor and start beta-blockers.
C. hCG overstimulation of the TSH receptor; manage the molar pregnancy.
D. Iodine deficiency; supplement with iodine and monitor.

A

C. hCG overstimulation of the TSH receptor; manage the molar pregnancy
Rationale: The primary cause of hyperthyroidism in gestational trophoblastic disease is hCG overstimulation of the TSH receptor, leading to increased thyroid hormone production. The key to management is treating the underlying molar pregnancy, which will help normalize the thyroid function.

How well did you know this?
1
Not at all
2
3
4
5
Perfectly
65
Q

A 27-year-old woman with hyperemesis gravidarum at 9 weeks gestation presents with symptoms of thyrotoxicosis. Blood tests show elevated free T4 and low TSH, and her hCG levels are markedly high.

Question: What is the expected outcome for her thyroid function by mid-pregnancy, and what treatment is needed at this time?

A. Thyroid function will normalize; no treatment needed.
B. Thyroid function will worsen; start PTU.
C. Thyroid function will remain abnormal; treat with methimazole.
D. Thyroid function will fluctuate; initiate radioactive iodine.

A

A. Thyroid function will normalize; no treatment needed
Rationale: In cases of gestational thyrotoxicosis associated with hyperemesis gravidarum, thyroid function typically normalizes by mid-pregnancy without the need for treatment.

How well did you know this?
1
Not at all
2
3
4
5
Perfectly
66
Q

What is the most common cause of hypothyroidism in pregnancy?

A. Graves’ disease
B. Iodine deficiency
C. Antithyroid peroxidase antibodies (anti-TPO)
D. Thyroid cancer

A

C. Antithyroid peroxidase antibodies (anti-TPO)
Rationale: Anti-TPO antibodies cause autoimmune destruction of the thyroid gland, commonly associated with Hashimoto’s thyroiditis, making it the leading cause of hypothyroidism in pregnancy.

How well did you know this?
1
Not at all
2
3
4
5
Perfectly
67
Q

Which of the following is the hallmark laboratory finding in subclinical hypothyroidism?

A. High TSH and low thyroxine (T4)
B. Normal TSH and high thyroxine (T4)
C. High TSH and normal thyroxine (T4)
D. Low TSH and low thyroxine (T4)

A

C. High TSH and normal thyroxine (T4)
Rationale: Subclinical hypothyroidism is characterized by elevated TSH with normal thyroxine levels (T4), indicating that the thyroid gland is slightly underactive but still producing adequate thyroid hormone.

Hypothyroidism: High TSH, low T₄ (and sometimes low T₃).
Hyperthyroidism: Low TSH, high T₄ and/or T₃.

How well did you know this?
1
Not at all
2
3
4
5
Perfectly
68
Q

Which of the following symptoms is NOT typically associated with overt hypothyroidism during pregnancy?

A. Fatigue
B. Weight loss
C. Cold intolerance
D. Constipation

A

B. Weight loss
Rationale: Weight loss is not typically seen in hypothyroidism. In contrast, hypothyroidism is more commonly associated with weight gain, fatigue, cold intolerance, and constipation.

How well did you know this?
1
Not at all
2
3
4
5
Perfectly
69
Q

Which form of hypothyroidism is more common in pregnancy?

A. Overt hypothyroidism
B. Subclinical hypothyroidism
C. Hashimoto’s thyroiditis
D. Postpartum thyroiditis

A

B. Subclinical hypothyroidism
Rationale: Subclinical hypothyroidism is more common than overt hypothyroidism in pregnancy, affecting more women (23 per 1000 pregnancies compared to 2-10 per 1000 for overt hypothyroidism).

How well did you know this?
1
Not at all
2
3
4
5
Perfectly
70
Q

What is the likely impact of untreated overt hypothyroidism on pregnancy outcomes?

A. No impact, the condition resolves postpartum
B. Infertility, miscarriage, or preterm labor
C. Fetal hyperthyroidism
D. Fetal macrocephaly

A

B. Infertility, miscarriage, or preterm labor
Rationale: Untreated overt hypothyroidism can lead to infertility, spontaneous abortion, and preterm labor due to the significant impact of thyroid hormones on pregnancy and fetal development.

How well did you know this?
1
Not at all
2
3
4
5
Perfectly
71
Q

Case Scenario:
A 30-year-old woman at 8 weeks gestation presents with fatigue, cold intolerance, and weight gain. She has a history of Hashimoto’s thyroiditis but stopped taking her levothyroxine 6 months ago. Her TSH is elevated, and her free thyroxine (T4) is low.

Question: What is the most appropriate next step in managing her condition?

A. Restart levothyroxine at a higher dose and monitor TSH.
B. No treatment needed as this will resolve postpartum.
C. Start propylthiouracil and repeat thyroid function tests.
D. Administer iodine supplements and follow up in 3 months.

A

A. Restart levothyroxine at a higher dose and monitor TSH
Rationale: The patient has overt hypothyroidism, confirmed by her high TSH and low T4. The appropriate treatment is to restart levothyroxine, ensuring adequate thyroid hormone levels to support pregnancy.

72
Q

A 28-year-old woman at 10 weeks gestation is found to have a TSH level of 7 mU/L and a normal free thyroxine (T4) level. She has no significant symptoms and no known history of thyroid disease.

Question: What is the diagnosis, and what is the recommended management?

A. Overt hypothyroidism; start levothyroxine immediately.
B. Subclinical hypothyroidism; consider levothyroxine and monitor TSH.
C. Euthyroid; no treatment is needed.
D. Hyperthyroidism; start methimazole.

A

B. Subclinical hypothyroidism; consider levothyroxine and monitor TSH
Rationale: The patient’s elevated TSH and normal T4 are consistent with subclinical hypothyroidism. Levothyroxine may be considered, especially in pregnancy, to prevent complications, and TSH should be monitored regularly.

73
Q

A 32-year-old woman with a history of infertility presents at 12 weeks gestation after successful IVF. Her lab results show a TSH level of 8 mU/L and low free thyroxine (T4). She reports feeling tired, constipated, and cold.

Question: What is the most likely diagnosis, and what complication is this patient at risk for if untreated?

A. Overt hypothyroidism; risk of miscarriage or preterm labor
B. Subclinical hypothyroidism; no complications expected
C. Euthyroid; risk of fetal hyperthyroidism
D. Postpartum thyroiditis; risk of preeclampsia

A

A. Overt hypothyroidism; risk of miscarriage or preterm labor
Rationale: The patient has overt hypothyroidism, as indicated by her high TSH and low T4 levels, along with classic hypothyroid symptoms. If untreated, she is at risk for miscarriage, preterm labor, and other pregnancy complications.

74
Q

A 35-year-old woman in her first trimester presents with mild fatigue and muscle cramps. Her TSH is 6.5 mU/L, but her free thyroxine (T4) is within normal limits. She has no other significant symptoms.

Question: What is the best course of action for this patient’s thyroid function during pregnancy?

A. No treatment needed, recheck thyroid function postpartum.
B. Start levothyroxine and monitor TSH during pregnancy.
C. Start iodine supplements and monitor symptoms.
D. Administer methimazole and repeat thyroid function tests.

A

B. Start levothyroxine and monitor TSH during pregnancy
Rationale: The patient likely has subclinical hypothyroidism (elevated TSH with normal T4). During pregnancy, treatment with levothyroxine is recommended to avoid potential complications like preterm birth and impaired fetal development.

75
Q

A 29-year-old woman at 14 weeks gestation presents for a routine checkup. Her thyroid function tests show a TSH of 4.8 mU/L and normal T4. She denies any symptoms of hypothyroidism.

Question: How should her thyroid function be managed during the remainder of her pregnancy?

A. Recheck thyroid function in 6-8 weeks, initiate levothyroxine if TSH rises.
B. No further testing needed as her levels are normal.
C. Start methimazole immediately and monitor symptoms.
D. Administer iodine supplements and recheck thyroid function in the third trimester.

A

A. Recheck thyroid function in 6-8 weeks, initiate levothyroxine if TSH rises
Rationale: The patient’s mildly elevated TSH with normal T4 is consistent with subclinical hypothyroidism. It is important to monitor thyroid function during pregnancy, as rising TSH levels may warrant levothyroxine treatment to prevent complications.

76
Q

Which of the following is a key risk factor for developing subclinical hypothyroidism?

A. Type 1 diabetes mellitus
B. Type 2 diabetes mellitus
C. Vitamin D deficiency
D. High BMI

A

A. Type 1 diabetes mellitus
Rationale: Type 1 diabetes mellitus is associated with autoimmune conditions, including subclinical hypothyroidism. Autoimmune diseases often coexist, increasing the risk.

77
Q

At what serum TSH level should thyroxine treatment be initiated in women with subclinical hypothyroidism?

A. TSH >5 mU/L
B. TSH >8 mU/L
C. TSH >10 mU/L
D. TSH >15 mU/L

A

C. TSH >10 mU/L
Rationale: In cases of subclinical hypothyroidism, treatment with thyroxine is generally initiated when TSH exceeds 10 mU/L.

78
Q

What is the main reason why pregnancy increases the need for thyroxine in women with subclinical hypothyroidism?

A. Increased iodine metabolism
B. Increased estrogen production
C. Increased progesterone levels
D. Decreased placental function

A

B. Increased estrogen production
Rationale: Pregnancy increases the need for thyroxine due to increased estrogen production, which raises thyroid-binding globulin levels, thus increasing the demand for thyroid hormone.

79
Q

What fetal complication is most associated with untreated subclinical hypothyroidism during pregnancy?

A. Fetal hyperthyroidism
B. Preterm birth and low birth weight
C. Fetal macrosomia
D. Congenital adrenal hyperplasia

A

B. Preterm birth and low birth weight
Rationale: Untreated subclinical hypothyroidism in pregnancy is associated with a higher risk of preterm birth, low birth weight (LBW), and other adverse outcomes like placental abruption and stillbirths.

80
Q

A 28-year-old woman with a family history of hypothyroidism and Type 1 diabetes mellitus presents at 10 weeks of pregnancy for her first prenatal visit. She reports fatigue but has no other symptoms. Her TSH is 9.5 mU/L, and her free T4 is within normal limits.

Question: What is the most likely diagnosis, and what management is appropriate at this time?

A. Overt hypothyroidism; start levothyroxine immediately.
B. Subclinical hypothyroidism; monitor TSH levels and consider levothyroxine.
C. Euthyroid; no treatment is needed.
D. Hyperthyroidism; start methimazole.

A

B. Subclinical hypothyroidism; monitor TSH levels and consider levothyroxine
Rationale: This patient has subclinical hypothyroidism (elevated TSH, normal T4), and treatment should be considered, especially since she is pregnant, with TSH being close to the 10 mU/L threshold.

81
Q

A 30-year-old pregnant woman at 14 weeks gestation is diagnosed with subclinical hypothyroidism, and her TSH is 12 mU/L. She is started on levothyroxine therapy. Her doctor plans to monitor her thyroid function regularly during pregnancy.

Question: How should her levothyroxine dose be adjusted during pregnancy, and why?

A. Decrease the dose as pregnancy progresses.
B. Increase the dose by 25-50 ug/day every 4-6 weeks.
C. Keep the same dose throughout the pregnancy.
D. Stop the levothyroxine after 20 weeks of pregnancy.

A

B. Increase the dose by 25-50 ug/day every 4-6 weeks
Rationale: During pregnancy, thyroxine requirements increase due to elevated estrogen levels, so the dose of levothyroxine should be adjusted by 25-50 ug/day every 4-6 weeks to maintain normal thyroid hormone levels.

82
Q

A 32-year-old woman with a history of subclinical hypothyroidism is 16 weeks pregnant. She is on levothyroxine 100 ug/day. Her current thyroid function tests show a TSH of 3.5 mU/L and normal free T4. She is asymptomatic.

Question: What is the next best step in her management?

A. Increase the levothyroxine dose.
B. Continue the current levothyroxine dose and recheck TSH in 4-6 weeks.
C. Discontinue levothyroxine and recheck TSH postpartum.
D. Add iodine supplementation to her treatment.

A

B. Continue the current levothyroxine dose and recheck TSH in 4-6 weeks
Rationale: The patient’s current TSH and free T4 levels are within normal range, indicating that her current levothyroxine dose is adequate. TSH should be monitored every 4-6 weeks during pregnancy.

83
Q

A 29-year-old woman in her second trimester is diagnosed with subclinical hypothyroidism and started on levothyroxine. She asks if iodine supplementation is necessary during pregnancy.

Question: What advice should be given regarding iodine intake during pregnancy?

A. Iodine supplementation is not necessary.
B. Iodine supplementation is recommended at 220 ug/day.
C. Iodine should be avoided during pregnancy.
D. Increase dietary intake of iodine to 300 ug/day.

A

B. Iodine supplementation is recommended at 220 ug/day
Rationale: Iodine supplementation is essential during pregnancy, with a recommended daily intake of 220 ug/day to support thyroid function and prevent iodine deficiency-related complications.

84
Q

A 31-year-old woman with subclinical hypothyroidism becomes pregnant and is not taking any thyroid medications. She develops preeclampsia at 32 weeks and delivers a baby weighing 1.8 kg at 36 weeks. The baby is admitted to the NICU due to low birth weight.

Question: What is the likely cause of her pregnancy complications?

A. Her preeclampsia led to placental insufficiency and low birth weight.
B. Her untreated subclinical hypothyroidism contributed to preterm birth and low birth weight.
C. She has undiagnosed hyperthyroidism.
D. The complications are unrelated to her thyroid function.

A

B. Her untreated subclinical hypothyroidism contributed to preterm birth and low birth weight
Rationale: Untreated subclinical hypothyroidism during pregnancy is associated with preterm birth, low birth weight, and increased risk of NICU admissions. Thyroid function plays a significant role in fetal development and pregnancy outcomes.

85
Q

What is the most common cause of congenital hypothyroidism?

A. Thyroid agenesis or dyshormonogenesis
B. Iodine deficiency
C. Transient hypothyroidism
D. Maternal hypothyroidism

A

A. Thyroid agenesis or dyshormonogenesis
Rationale: Thyroid agenesis (absence of the thyroid gland) or dyshormonogenesis (abnormal thyroid hormone synthesis) accounts for 75% of cases of congenital hypothyroidism.

86
Q

What is the most important treatment for infants diagnosed with congenital hypothyroidism?

A. Iodine supplementation
B. Levothyroxine replacement therapy
C. Radioiodine therapy
D. Observation and re-evaluation at 6 months

A

B. Levothyroxine replacement therapy
Rationale: Aggressive levothyroxine replacement therapy is critical in treating congenital hypothyroidism to prevent cognitive impairments and improve long-term developmental outcomes.

87
Q

Which group of infants is most at risk for transient hypothyroxinemia?

A. Full-term infants with maternal hypothyroidism
B. Preterm infants
C. Infants with a family history of thyroid disease
D. Infants born with birth weight >4 kg

A

B. Preterm infants
Rationale: Preterm infants are at risk for transient hypothyroxinemia, a temporary decrease in thyroxine (T4) levels that often resolves without requiring long-term treatment.

88
Q

A newborn infant is diagnosed with congenital hypothyroidism after newborn screening revealed elevated TSH and low free T4. Further investigation shows that the infant has thyroid agenesis.

Question: What is the most appropriate next step in managing this infant’s thyroid condition?

A. Start iodine supplementation.
B. Administer levothyroxine and monitor TSH and free T4 levels.
C. Observe without treatment and repeat screening in 6 months.
D. Delay treatment until further diagnostic tests are completed.

A

B. Administer levothyroxine and monitor TSH and free T4 levels
Rationale: The infant has congenital hypothyroidism due to thyroid agenesis, and the recommended treatment is levothyroxine replacement therapy to normalize thyroid hormone levels and promote normal cognitive development.

89
Q

A premature infant born at 28 weeks gestation is diagnosed with transient hypothyroxinemia after screening shows low free T4 but normal TSH levels. The baby is otherwise healthy and feeding well.

Question: What is the most appropriate management for this infant?

A. Start levothyroxine replacement therapy.
B. Administer iodine supplementation.
C. Monitor thyroid function and expect resolution without treatment.
D. Perform thyroid ultrasound to assess for thyroid agenesis.

A

C. Monitor thyroid function and expect resolution without treatment
Rationale: Transient hypothyroxinemia in preterm infants typically resolves on its own. These infants should be monitored rather than treated unless thyroid function worsens or symptoms develop.

90
Q

A full-term newborn with normal birth weight is diagnosed with transient hypothyroidism after testing reveals a slightly elevated TSH and low T4. The condition is expected to resolve within a few months.

Question: What is the most likely cause of this transient hypothyroidism in the newborn?

A. Thyroid agenesis
B. Iodine deficiency in the mother
C. Placental transfer of maternal thyroid-blocking antibodies
D. Dyshormonogenesis

A

C. Placental transfer of maternal thyroid-blocking antibodies
Rationale: Transient hypothyroidism in newborns can be caused by the placental transfer of maternal thyroid-blocking antibodies, which may temporarily inhibit thyroid function but typically resolves over time.

91
Q

A baby is born with multiple congenital anomalies, including a cleft palate and heart defects. Screening shows elevated TSH and low T4 levels, and the baby is diagnosed with congenital hypothyroidism.

Question: What percentage of congenital hypothyroidism cases are associated with major anomalies like this?

A. 2%
B. 5%
C. 8%
D. 10%

A

C. 8%
Rationale: Congenital hypothyroidism is associated with major anomalies in about 8% of cases, and these infants may have additional congenital defects, requiring a multidisciplinary approach to care.

92
Q

What is the most common underlying cause of postpartum thyroiditis?

A. Iodine deficiency
B. Fetal microchimerism
C. Destructive lymphocytic thyroiditis
D. Viral infection

A

C. Destructive lymphocytic thyroiditis
Rationale: Postpartum thyroiditis is primarily caused by destructive lymphocytic thyroiditis, often triggered by an increase in thyroid autoantibodies.

93
Q

Which of the following is NOT a common symptom of postpartum thyroiditis?

A. Depression
B. Palpitations
C. Weight loss
D. Memory impairment

A

C. Weight loss
Rationale: Weight loss is typically associated with hyperthyroidism but is not a common symptom of postpartum thyroiditis. Symptoms include depression, palpitations, and memory impairment.

94
Q

What is the primary treatment for hyperthyroid symptoms in postpartum thyroiditis?

A. Levothyroxine
B. Methimazole
C. Beta-blockers
D. Radioactive iodine

A

C. Beta-blockers
Rationale: Beta-blockers are commonly used to manage the hyperthyroid symptoms of postpartum thyroiditis, such as palpitations and anxiety.

95
Q

A 32-year-old woman, 4 months postpartum, presents with fatigue, palpitations, and depression. Physical examination reveals a small, painless goiter. Thyroid function tests show a low TSH and elevated free T4. She has a history of Type 1 diabetes.

Question: What is the most likely diagnosis, and what is the appropriate initial management?

A. Graves’ disease; start methimazole.
B. Postpartum thyroiditis; treat with beta-blockers for symptoms.
C. Subclinical hyperthyroidism; no treatment needed.
D. Postpartum thyroiditis; start levothyroxine.

A

B. Postpartum thyroiditis; treat with beta-blockers for symptoms
Rationale: This patient has postpartum thyroiditis, which can cause hyperthyroid symptoms in the early phase. Beta-blockers are the first-line treatment to manage symptoms like palpitations.

96
Q

A 29-year-old woman, 6 months postpartum, presents with memory impairment, fatigue, and a feeling of carelessness. She was diagnosed with postpartum thyroiditis 2 months ago and was treated with beta-blockers for palpitations. Thyroid function tests now show elevated TSH and low free T4.

Question: What is the next step in management?

A. Continue beta-blockers.
B. Start levothyroxine for hypothyroidism.
C. Start methimazole for hyperthyroidism.
D. Monitor thyroid function and reassess in 6 months.

A

B. Start levothyroxine for hypothyroidism
Rationale: The patient has transitioned to the hypothyroid phase of postpartum thyroiditis. Levothyroxine is indicated to treat hypothyroidism, particularly given her symptoms of fatigue and cognitive changes.

97
Q

A 30-year-old woman with Type 1 diabetes presents with postpartum thyroiditis 4 months after delivery. She is currently experiencing palpitations and anxiety. Her thyroid function tests show low TSH and high free T4.

Question: What is the most likely underlying cause of her thyroid condition?

A. Destructive lymphocytic thyroiditis
B. Graves’ disease
C. Iodine deficiency
D. Thyroid adenoma

A

A. Destructive lymphocytic thyroiditis
Rationale: Postpartum thyroiditis is typically caused by destructive lymphocytic thyroiditis, which is associated with autoimmune conditions such as Type 1 diabetes.

98
Q

What is the primary imaging modality used to evaluate thyroid nodules in pregnancy?

A. CT scan
B. Ultrasound
C. MRI
D. Radioiodine scanning

A

B. Ultrasound
Rationale: Ultrasound is the preferred diagnostic tool for evaluating thyroid nodules during pregnancy because it is safe and provides detailed imaging. Radioiodine scanning is contraindicated due to the potential risks to the fetus.

99
Q

When is fine needle aspiration (FNA) of a thyroid nodule typically recommended during pregnancy?

A. Before mid-pregnancy
B. During the third trimester
C. After delivery
D. During labor

A

A. Before mid-pregnancy
Rationale: Fine needle aspiration (FNA) is best performed before mid-pregnancy to evaluate suspicious thyroid nodules and determine if they are benign or malignant.

100
Q

What is the recommended timing for thyroidectomy in pregnant women if a thyroid nodule is found to be malignant?

A. First trimester
B. 24-26 weeks gestation
C. 32-34 weeks gestation
D. Postpartum

A

B. 24-26 weeks gestation
Rationale: Thyroidectomy for a confirmed malignant thyroid nodule should be performed during the 24-26 weeks gestation period, as this is considered the safest window for surgery during pregnancy.

101
Q

What size of a cystic thyroid lesion warrants further evaluation in pregnancy?

A. 1 cm
B. 2 cm
C. 4 cm
D. 5 cm

A

C. 4 cm
Rationale: Cystic thyroid lesions greater than 4 cm should be evaluated further to assess the risk of malignancy, although cystic lesions are less likely to be malignant compared to solid ones.

102
Q

A 30-year-old woman in her third trimester is experiencing fatigue and mild muscle cramps. Laboratory studies reveal low serum calcium levels. Her 1,25-dihydroxyvitamin D levels are elevated. She has no other symptoms, and her pregnancy is otherwise progressing normally.

Question: What physiological change in pregnancy is likely contributing to her elevated vitamin D levels?

A. Increased renal excretion of calcium
B. Two-fold increase in 1,25-dihydroxyvitamin D to increase calcium absorption
C. Reduced bone resorption due to low PTH levels
D. Elevated calcitonin levels reducing calcium absorption

A

B. Two-fold increase in 1,25-dihydroxyvitamin D to increase calcium absorption
Rationale: During pregnancy, 1,25-dihydroxyvitamin D levels increase two-fold to enhance gastrointestinal calcium absorption to meet the growing fetal demands for calcium.

103
Q

A pregnant woman at 34 weeks gestation is found to have a low calcium level during routine blood work. Her healthcare provider notes that her body is losing more calcium through urine due to an increased glomerular filtration rate (GFR).

Question: What compensatory mechanism helps meet the body’s increased calcium needs during pregnancy?

A. Increased PTH levels to reduce renal calcium loss
B. Decreased calcitonin levels to enhance bone resorption
C. Increased intestinal absorption of calcium due to elevated 1,25-dihydroxyvitamin D
D. Decreased bone resorption due to elevated vitamin D

A

C. Increased intestinal absorption of calcium due to elevated 1,25-dihydroxyvitamin D
Rationale: During pregnancy, 1,25-dihydroxyvitamin D levels increase to improve intestinal absorption of calcium, which helps compensate for increased calcium loss through the kidneys.

104
Q

A 28-year-old woman is in her third trimester and is concerned about her low calcium intake. She learns that the fetus requires a substantial amount of calcium, especially in the final months of pregnancy.

Question: How much calcium does the fetus require daily in late pregnancy, and what can the mother do to meet this need?

A. 100 mg/day; increase dietary phosphorus
B. 300 mg/day; increase dietary calcium and ensure adequate vitamin D intake
C. 500 mg/day; take calcium supplements only
D. 200 mg/day; reduce PTH levels with medication

A

B. 300 mg/day; increase dietary calcium and ensure adequate vitamin D intake
Rationale: The fetus requires 300 mg/day of calcium in late pregnancy. The mother can meet this demand by increasing dietary calcium intake and ensuring she gets enough vitamin D for optimal calcium absorption.

105
Q

A 35-year-old pregnant woman is diagnosed with hypercalcemia in the third trimester. Blood tests show elevated calcitonin levels and suppressed PTH levels. Her doctor suspects a condition affecting calcium regulation.

Question: What role does calcitonin play in regulating calcium levels during pregnancy?

A. Stimulates calcium release from bones
B. Increases calcium absorption in the intestines
C. Acts as a hypocalcemic hormone by opposing PTH
D. Increases renal reabsorption of calcium

A

C. Acts as a hypocalcemic hormone by opposing PTH
Rationale: Calcitonin is a hypocalcemic hormone that opposes the action of PTH by inhibiting bone resorption and lowering blood calcium levels, especially during pregnancy.

106
Q

A pregnant woman at 30 weeks gestation is concerned about the impact of her low calcium levels on her baby. Her doctor explains that, despite her low calcium intake, the fetus still receives sufficient calcium for bone development.

Question: How does the maternal body ensure that the fetus receives adequate calcium during pregnancy?

A. The fetus directly regulates maternal PTH levels.
B. Increased levels of 1,25-dihydroxyvitamin D increase maternal calcium absorption.
C. The placenta produces calcium to supplement fetal needs.
D. Maternal bone resorption increases in response to calcitonin.

A

B. Increased levels of 1,25-dihydroxyvitamin D increase maternal calcium absorption
Rationale: During pregnancy, 1,25-dihydroxyvitamin D levels increase, enhancing calcium absorption from the gastrointestinal tract to ensure that the fetus receives sufficient calcium for proper development.

107
Q

What is the primary cause of hypercalcemia in 90% of cases of hyperparathyroidism?

A. Renal failure
B. Primary hyperparathyroidism or cancer
C. Vitamin D deficiency
D. Hypoparathyroidism

A

B. Primary hyperparathyroidism or cancer
Rationale: Hypercalcemia in 90% of cases is due to primary hyperparathyroidism or malignancy. These conditions increase the levels of circulating calcium by increasing PTH or tumor-secreted factors that mimic PTH.

108
Q

Which of the following is NOT a common symptom of hypercalcemia in hyperparathyroidism?

A. Renal calculi
B. Stupor
C. Pancreatitis
D. Weight gain

A

D. Weight gain
Rationale: Common symptoms of hypercalcemia include renal calculi, stupor, pancreatitis, nausea, and vomiting. Weight gain is not typically associated with hyperparathyroidism.

109
Q

Which pregnancy complication is associated with hyperparathyroidism?

A. Fetal macrosomia
B. Neonatal tetany
C. Placental abruption
D. Gestational diabetes

A

B. Neonatal tetany
Rationale: Hyperparathyroidism can lead to neonatal tetany due to severe hypocalcemia in the newborn, as well as complications such as stillbirth and preterm delivery.

110
Q

When is surgical excision recommended in hyperparathyroidism during pregnancy?

A. Serum calcium is greater than 0.5 mg/dL above normal values.
B. Serum calcium is greater than 1.0 mg/dL above normal values.
C. Serum calcium is below normal levels.
D. Serum calcium is within normal limits.

A

B. Serum calcium is greater than 1.0 mg/dL above normal values
Rationale: Surgical excision is indicated when serum calcium levels are greater than 1.0 mg/dL over normal values or if the patient has symptomatic adenomas.

111
Q

What is the recommended emergency treatment for a hypercalcemic crisis in hyperparathyroidism?

A. Diuresis with normal saline or furosemide
B. Methimazole therapy
C. Immediate thyroidectomy
D. Oral calcium supplementation

A

A. Diuresis with normal saline or furosemide
Rationale: In cases of hypercalcemic crisis, emergency treatment includes diuresis with normal saline or furosemide to lower calcium levels and prevent complications such as stupor or renal failure.

112
Q

Case Scenario:
A 33-year-old pregnant woman at 26 weeks gestation presents with nausea, vomiting, and fatigue. Laboratory results show elevated serum calcium and parathyroid hormone (PTH) levels. Ultrasound of the kidneys shows the presence of renal calculi. She has a history of hyperparathyroidism.

Question: What is the most likely diagnosis, and what is the appropriate treatment for her hypercalcemia?

A. Hypercalcemia due to vitamin D deficiency; treat with vitamin D supplements.
B. Hypercalcemia due to hyperparathyroidism; initiate diuresis with normal saline or furosemide.
C. Hypocalcemia; treat with calcium supplements.
D. Hyperthyroidism; treat with beta-blockers.

A

B. Hypercalcemia due to hyperparathyroidism; initiate diuresis with normal saline or furosemide
Rationale: The patient’s elevated calcium and PTH levels along with renal calculi indicate hypercalcemia due to hyperparathyroidism. Diuresis with normal saline or furosemide is the appropriate treatment to lower calcium levels.

113
Q

A 30-year-old woman, 32 weeks pregnant, presents with weakness, dehydration, and stupor. Her serum calcium is significantly elevated, and she is diagnosed with a hypercalcemic crisis due to hyperparathyroidism.

Question: What immediate treatment should be administered to lower her calcium levels and stabilize her condition?

A. Oral phosphate
B. Emergency thyroidectomy
C. Diuresis with normal saline or furosemide
D. Levothyroxine

A

C. Diuresis with normal saline or furosemide
Rationale: The patient is in a hypercalcemic crisis, requiring urgent diuresis with normal saline or furosemide to reduce her calcium levels and stabilize her condition.

114
Q

A 28-year-old pregnant woman at 24 weeks gestation presents with mild hypercalcemia and is asymptomatic. Her serum calcium is 1.2 mg/dL above normal values. She is diagnosed with hyperparathyroidism.

Question: What is the most appropriate next step in managing her condition?

A. Immediate surgical excision of the parathyroid adenoma
B. Oral phosphate supplementation and surgery after delivery
C. Methimazole therapy for hyperthyroidism
D. Oral calcium supplementation

A

B. Oral phosphate supplementation and surgery after delivery
Rationale: In asymptomatic patients with mild hypercalcemia, oral phosphate supplementation can be used to manage calcium levels, and surgery can be postponed until after delivery unless the patient becomes symptomatic.

115
Q

A pregnant woman with hyperparathyroidism presents with severe nausea, vomiting, and dehydration. Her serum calcium is elevated, and she is at risk for a hypercalcemic crisis. Her doctor is concerned about potential complications.

Question: What pregnancy complications could arise from untreated hyperparathyroidism?

A. Preterm delivery and neonatal tetany
B. Placenta previa and gestational diabetes
C. Fetal macrosomia and hypertension
D. Intrauterine growth restriction and oligohydramnios

A

A. Preterm delivery and neonatal tetany
Rationale: Hyperparathyroidism during pregnancy increases the risk of preterm delivery, stillbirth, and neonatal tetany due to severe hypocalcemia in the neonate.

116
Q

A 34-year-old pregnant woman at 28 weeks gestation is diagnosed with hyperparathyroidism. Her serum calcium is elevated, and she has mild renal symptoms but no other significant issues. Her physician recommends surgical excision of her parathyroid adenoma.

Question: When is the optimal time for surgical intervention in this patient?

A. Immediately, regardless of gestational age
B. During the first trimester
C. Between 24-26 weeks gestation
D. After delivery

A

C. Between 24-26 weeks gestation
Rationale: If surgery is necessary for hyperparathyroidism, the safest time for surgical excision during pregnancy is between 24-26 weeks gestation, minimizing risk to both the mother and fetus.

117
Q

What is the most common cause of hypoparathyroidism?

A. Autoimmune disease
B. Parathyroid or thyroid surgery
C. Vitamin D deficiency
D. Chronic kidney disease

A

B. Parathyroid or thyroid surgery
Rationale: The most common cause of hypoparathyroidism is accidental damage or removal of the parathyroid glands during parathyroid or thyroid surgery, leading to insufficient PTH production.

118
Q

What complication can occur in the fetus of a mother with hypoparathyroidism?

A. Fetal macrosomia
B. Skeletal demineralization
C. Hypoglycemia
D. Neonatal hyperparathyroidism

A

B. Skeletal demineralization
Rationale: Fetal skeletal demineralization can occur in cases of maternal hypoparathyroidism due to impaired calcium regulation and hypocalcemia.

119
Q

Which of the following is part of the treatment for hypoparathyroidism?

A. Levothyroxine
B. Calcitonin
C. 1,25-dihydroxyvitamin D3
D. Radioiodine therapy

A

C. 1,25-dihydroxyvitamin D3
Rationale: 1,25-dihydroxyvitamin D3 is the active form of vitamin D used to enhance calcium absorption in patients with hypoparathyroidism.

120
Q

Why is a diet low in phosphates recommended in hypoparathyroidism?

A. To prevent hyperphosphatemia, which can worsen hypocalcemia
B. To increase calcium excretion from the kidneys
C. To prevent bone demineralization
D. To reduce the risk of kidney stones

A

A. To prevent hyperphosphatemia, which can worsen hypocalcemia
Rationale: A low-phosphate diet is recommended to prevent hyperphosphatemia

121
Q

Which of the following is a key component of managing hypoparathyroidism during pregnancy?

A. Methimazole
B. Radioactive iodine therapy
C. Large doses of vitamin D
D. Beta-blockers

A

C. Large doses of vitamin D
Rationale: Patients with hypoparathyroidism require large doses of vitamin D to maintain calcium levels due to the lack of PTH.

122
Q

A 34-year-old woman who had thyroid surgery six months ago presents with symptoms of muscle cramps, fatigue, and tingling in her hands. Blood tests reveal low calcium and low PTH levels. She is currently pregnant at 20 weeks gestation.

Question: What is the most likely diagnosis, and what is the appropriate treatment?

A. Hypocalcemia due to hypothyroidism; treat with levothyroxine.
B. Hypoparathyroidism due to thyroid surgery; treat with 1,25-dihydroxyvitamin D3 and calcium.
C. Hyperparathyroidism; treat with surgery.
D. Hypocalcemia due to vitamin D deficiency; treat with oral vitamin D supplements.

A

B. Hypoparathyroidism due to thyroid surgery; treat with 1,25-dihydroxyvitamin D3 and calcium
Rationale: The patient has hypoparathyroidism caused by thyroid surgery, leading to hypocalcemia. The treatment includes 1,25-dihydroxyvitamin D3 and calcium supplementation to correct calcium levels.

123
Q

A pregnant woman at 28 weeks gestation presents with low calcium levels and a history of parathyroid surgery. Her doctor diagnoses her with hypoparathyroidism and prescribes calcium and vitamin D supplements.

Question: Why is it important for this patient to follow a diet low in phosphates?

A. To prevent kidney stones
B. To enhance vitamin D absorption
C. To prevent hyperphosphatemia, which can worsen hypocalcemia
D. To increase bone density

A

C. To prevent hyperphosphatemia, which can worsen hypocalcemia
Rationale: A low-phosphate diet helps prevent hyperphosphatemia, which can worsen hypocalcemia in patients with hypoparathyroidism, as phosphate competes with calcium.

124
Q

A 30-year-old woman who recently had thyroid surgery is diagnosed with hypoparathyroidism. She is 24 weeks pregnant and concerned about the potential effects of her condition on her baby.

Question: What is a potential complication for the fetus if the mother’s hypoparathyroidism is not well managed?

A. Fetal macrosomia
B. Skeletal demineralization
C. Neonatal hyperparathyroidism
D. Fetal hypothyroidism

A

B. Skeletal demineralization
Rationale: Skeletal demineralization can occur in the fetus if the mother’s hypoparathyroidism is not well controlled, leading to insufficient calcium for fetal bone development.

125
Q

A 35-year-old pregnant woman presents with muscle cramps and low calcium levels. She is diagnosed with hypoparathyroidism and started on 1,25-dihydroxyvitamin D3 and calcium supplementation.

Question: What is the primary role of 1,25-dihydroxyvitamin D3 in managing hypoparathyroidism?

A. It increases renal calcium excretion.
B. It enhances gastrointestinal calcium absorption.
C. It increases phosphate excretion.
D. It decreases PTH production.

A

B. It enhances gastrointestinal calcium absorption
Rationale: 1,25-dihydroxyvitamin D3 plays a critical role in enhancing gastrointestinal calcium absorption, which is essential for managing hypocalcemia in patients with hypoparathyroidism.

126
Q

A woman with hypoparathyroidism due to previous thyroid surgery is 26 weeks pregnant. She is asymptomatic but has slightly low calcium levels. Her doctor is considering adjusting her treatment.

Question: What is the most appropriate adjustment to her treatment regimen?

A. Increase her calcium and vitamin D supplementation.
B. Discontinue calcium supplementation.
C. Add oral phosphate supplements.
D. Start her on calcitonin therapy.

A

A. Increase her calcium and vitamin D supplementation
Rationale: In hypoparathyroidism, maintaining adequate calcium and vitamin D levels is essential, especially during pregnancy. Increasing supplementation helps manage slight hypocalcemia and prevent complications.

127
Q

What is a common complication of pregnancy-induced osteoporosis?

A. Hypertension
B. Fractures
C. Weight gain
D. Hypercalcemia

A

B. Fractures
Rationale: Pregnancy-induced osteoporosis often results in bone fractures, especially in weight-bearing bones, due to reduced bone density.

128
Q

Which of the following is a risk factor for pregnancy-induced osteoporosis?

A. Prolonged bed rest
B. Vitamin D toxicity
C. Low calcium intake
D. High potassium intake

A

A. Prolonged bed rest
Rationale: Prolonged bed rest reduces mechanical stress on bones, leading to decreased bone density and increasing the risk of pregnancy-induced osteoporosis.

Causes
Heparin use: Long-term use of heparin, particularly during pregnancy, can contribute to osteoporosis.
Prolonged bed rest: Immobility can result in decreased bone density, leading to osteoporosis.
Corticosteroid therapy: Long-term corticosteroid use is a well-known risk factor for osteoporosis.

129
Q

Which medication is associated with an increased risk of pregnancy-induced osteoporosis?

A. Insulin
B. Heparin
C. Metformin
D. Levothyroxine

A

B. Heparin
Rationale: Long-term heparin use during pregnancy is associated with a higher risk of osteoporosis and bone fractures due to its effects on bone density.

130
Q

Idiopathic osteonecrosis of the femoral head during pregnancy is commonly associated with which condition?

A. Hypertension
B. Sickle cell anemia
C. Type 1 diabetes
D. Preeclampsia

A

B. Sickle cell anemia
Rationale: Idiopathic osteonecrosis of the femoral head is often associated with sickle cell anemia, which can lead to poor blood supply to the bone and contribute to necrosis.

131
Q

What is the primary treatment for pregnancy-induced osteoporosis?

A. Calcium and vitamin D supplementation
B. Beta-blockers
C. Methotrexate
D. Insulin therapy

A

A. Calcium and vitamin D supplementation
Rationale: The primary treatment for pregnancy-induced osteoporosis includes calcium and vitamin D supplementation to support bone health and prevent further bone loss.

132
Q

Case Scenario:
A 30-year-old pregnant woman presents with back pain and a recent fracture of her rib. She has been on prolonged bed rest due to complications with her pregnancy. Her calcium levels are normal, and she has no history of trauma.

Question: What is the most likely diagnosis, and what is the appropriate treatment?

A. Pregnancy-induced osteoporosis; treat with calcium and vitamin D supplementation
B. Hyperparathyroidism; treat with calcitonin
C. Osteoarthritis; treat with NSAIDs
D. Sickle cell anemia; treat with blood transfusions

A

A. Pregnancy-induced osteoporosis; treat with calcium and vitamin D supplementation
Rationale: The patient’s fracture and back pain, along with her prolonged bed rest, suggest pregnancy-induced osteoporosis. Calcium and vitamin D supplementation is the appropriate treatment to support bone health.

133
Q

A pregnant woman is diagnosed with idiopathic osteonecrosis of the femoral head. She also has a history of sickle cell anemia. She presents with hip pain that worsens with weight-bearing activity.

Question: What is the underlying condition likely contributing to her osteonecrosis, and how should she be managed?

A. Vitamin D deficiency; start vitamin D supplements
B. Sickle cell anemia; treat with calcium and vitamin D
C. Hypoparathyroidism; treat with PTH replacement
D. Hyperparathyroidism; treat with surgery

A

B. Sickle cell anemia; treat with calcium and vitamin D
Rationale: The patient’s sickle cell anemia is contributing to the development of idiopathic osteonecrosis of the femoral head. Treatment should include calcium and vitamin D to prevent further bone loss.

134
Q

A 28-year-old pregnant woman is undergoing heparin therapy due to a history of deep vein thrombosis. She presents with worsening bone pain and has been diagnosed with pregnancy-induced osteoporosis.

Question: What is the most likely cause of her osteoporosis, and what is the recommended treatment?

A. Prolonged bed rest; start physiotherapy
B. Heparin use; start calcium and vitamin D supplementation
C. Corticosteroid therapy; reduce steroid dosage
D. Thyroid dysfunction; start levothyroxine

A

B. Heparin use; start calcium and vitamin D supplementation
Rationale: Heparin use is a risk factor for pregnancy-induced osteoporosis. The recommended treatment is calcium and vitamin D supplementation to mitigate bone loss.

135
Q

A pregnant woman presents with pain in her lower back and hips. She has a history of corticosteroid therapy for a chronic condition. Imaging reveals multiple vertebral fractures. Her physician suspects pregnancy-induced osteoporosis.

Question: What is the most likely contributing factor to her osteoporosis, and how should she be treated?

A. Corticosteroid therapy; treat with calcium and vitamin D supplementation
B. Hyperthyroidism; treat with methimazole
C. Vitamin D deficiency; treat with high-dose vitamin D
D. Gestational diabetes; treat with insulin

A

A. Corticosteroid therapy; treat with calcium and vitamin D supplementation
Rationale: Corticosteroid therapy is a known risk factor for osteoporosis, especially in pregnant women. Treatment should include calcium and vitamin D to support bone health and prevent further fractures.

136
Q

A 32-year-old woman with sickle cell anemia presents with hip pain and difficulty walking during her pregnancy. Imaging confirms osteonecrosis of the femoral head.

Question: What treatment should be initiated to manage her bone health during pregnancy?

A. High-dose steroids
B. Calcium and vitamin D supplementation
C. Heparin therapy
D. Chemotherapy

A

B. Calcium and vitamin D supplementation
Rationale: In cases of osteonecrosis related to sickle cell anemia, treatment with calcium and vitamin D is essential to support bone health and reduce the risk of further complications.

137
Q

Which hormone is increased during pregnancy due to secretion from the placenta?

A. Corticotropin-releasing hormone (CRH)
B. Progesterone
C. Oxytocin
D. Insulin

A

A. Corticotropin-releasing hormone (CRH)
Rationale: The placenta secretes CRH during pregnancy, which leads to an increase in serum corticotropin levels.

138
Q

What is the most common dangerous adrenal gland disorder during pregnancy?

A. Hyperaldosteronism
B. Pheochromocytoma
C. Cushing’s syndrome
D. Addison’s disease

A

B. Pheochromocytoma
Rationale: Pheochromocytoma, a chromaffin tumor that secretes catecholamines (e.g., adrenaline), is particularly dangerous during pregnancy due to its association with hypertensive crises.

139
Q

Which of the following is NOT a typical symptom of pheochromocytoma during pregnancy?

A. Hypertensive crisis
B. Seizure disorders
C. Anxiety attacks
D. Hypotension

A

D. Hypotension
Rationale: Hypotension is not a common symptom of pheochromocytoma. Instead, hypertensive crises, seizures, and anxiety attacks are frequent due to excess catecholamine secretion.

140
Q

What is the primary diagnostic test for pheochromocytoma during pregnancy?

A. CT scan
B. MRI
C. 24-hour urine catecholamine metabolite quantification
D. Ultrasound

A

C. 24-hour urine catecholamine metabolite quantification
Rationale: During pregnancy, 24-hour urine catecholamine metabolite quantification is the preferred test to diagnose pheochromocytoma, as imaging methods like CT and MRI are avoided.

141
Q

What is the recommended surgical treatment timing for pheochromocytoma during pregnancy?

A. 10-12 weeks gestation
B. 16-20 weeks gestation
C. 24-28 weeks gestation
D. After delivery

A

B. 16-20 weeks gestation
Rationale: Laparoscopic adrenalectomy is recommended between 16-20 weeks gestation in cases of pheochromocytoma to manage the tumor and avoid complications later in pregnancy.

142
Q

A 28-year-old pregnant woman presents at 18 weeks gestation with sudden, severe headaches, anxiety, and high blood pressure. She has experienced similar episodes intermittently for several weeks. Her physician suspects pheochromocytoma.

Question: What is the most appropriate diagnostic test for confirming pheochromocytoma in this pregnant patient?

A. CT scan of the abdomen
B. MRI of the adrenal glands
C. 24-hour urine catecholamine metabolite quantification
D. X-ray

A

C. 24-hour urine catecholamine metabolite quantification
Rationale: 24-hour urine catecholamine metabolite quantification is the safest and most accurate diagnostic test for pheochromocytoma during pregnancy. Imaging such as CT and MRI is avoided due to risks to the fetus.

143
Q

A 32-year-old woman at 20 weeks gestation presents with episodes of severe hypertension, palpitations, and anxiety. Lab results show elevated urine catecholamine levels, confirming pheochromocytoma. The patient is concerned about the risks of surgery during pregnancy.

Question: What is the recommended treatment for this patient’s pheochromocytoma at this gestational age?

A. Laparoscopic adrenalectomy
B. Immediate delivery
C. Beta-blockers only
D. Antihypertensive therapy without surgery

A

A. Laparoscopic adrenalectomy
Rationale: Laparoscopic adrenalectomy between 16-20 weeks gestation is the recommended treatment for pheochromocytoma to remove the tumor and prevent hypertensive crises, which pose significant risks to both the mother and fetus.

144
Q

A pregnant woman at 24 weeks gestation is diagnosed with pheochromocytoma after experiencing repeated hypertensive crises and seizures. Her physician decides to manage her condition medically until delivery.

Question: What class of medication should be used to manage this patient’s hypertension caused by pheochromocytoma?

A. Alpha-adrenergic blockers
B. Calcium channel blockers
C. Diuretics
D. ACE inhibitors

A

A. Alpha-adrenergic blockers
Rationale: Alpha-adrenergic blockers are the first-line treatment for controlling hypertension in patients with pheochromocytoma. These medications help to counteract the effects of excess catecholamines.

145
Q

A 29-year-old woman at 19 weeks gestation presents with episodic hypertension, headaches, and anxiety. Urine catecholamine testing is positive for pheochromocytoma. The patient is started on an alpha-adrenergic blocker but still experiences significant symptoms.

Question: What additional medication can be added to improve control of this patient’s symptoms?

A. Beta-blockers
B. ACE inhibitors
C. Methimazole
D. Diuretics

A

A. Beta-blockers
Rationale: After initiating alpha-adrenergic blockers, beta-blockers can be added to further control heart rate and reduce the risk of hypertensive episodes in patients with pheochromocytoma.

146
Q

What is the most common cause of Cushing syndrome in pregnancy?

A. Pituitary adenoma
B. Adrenal adenoma
C. Iatrogenic corticosteroid use
D. Ectopic ACTH-secreting tumor

A

C. Iatrogenic corticosteroid use
Rationale: Iatrogenic corticosteroid use (due to long-term treatment with corticosteroids) is the most common cause of Cushing syndrome in pregnancy.

147
Q

Which of the following is a typical clinical sign of Cushing syndrome?

A. Hypotension
B. Truncal obesity
C. Hyperpigmentation
D. Hypoglycemia

A

B. Truncal obesity
Rationale: Truncal obesity is a hallmark feature of Cushing syndrome, along with moon facies, buffalo hump, and cutaneous striae.

148
Q

What diagnostic test confirms the diagnosis of Cushing syndrome in pregnancy?

A. Serum ACTH levels
B. 24-hour urine free cortisol measurement
C. Serum calcium
D. Plasma renin activity

A

B. 24-hour urine free cortisol measurement
Rationale: The 24-hour urine free cortisol measurement is the standard diagnostic test to confirm Cushing syndrome, as it reflects excess cortisol production.

Diagnosis is confirmed by:
Elevated 24-hour urine free cortisol levels.
Elevated plasma cortisol that does not respond to dexamethasone suppression.

149
Q

What fetal complication is associated with untreated Cushing syndrome during pregnancy?

A. Fetal macrosomia
B. Neonatal hyperthyroidism
C. Preterm delivery
D. Congenital heart disease

A

C. Preterm delivery
Rationale: Untreated Cushing syndrome during pregnancy increases the risk of preterm delivery and high perinatal morbidity and mortality.

150
Q

What is the definitive treatment for Cushing syndrome caused by adrenal adenoma during pregnancy?

A. Chemotherapy
B. Antihypertensives
C. Adrenalectomy
D. Beta-blockers

A

C. Adrenalectomy
Rationale: The definitive treatment for Cushing syndrome caused by an adrenal adenoma is adrenalectomy, which removes the tumor causing excess cortisol production.

151
Q

A 32-year-old pregnant woman at 24 weeks gestation presents with weight gain, truncal obesity, moon facies, and purplish stretch marks. She has a history of corticosteroid use for a chronic autoimmune condition. Blood tests show elevated 24-hour urine free cortisol and plasma cortisol levels that are not suppressed by dexamethasone.

Question: What is the most likely diagnosis and the cause of her symptoms?

A. Hyperthyroidism due to Graves’ disease
B. Cushing syndrome due to iatrogenic corticosteroid use
C. Addison’s disease
D. Pheochromocytoma

A

B. Cushing syndrome due to iatrogenic corticosteroid use
Rationale: The patient’s signs (moon facies, truncal obesity, striae) and elevated cortisol levels are consistent with Cushing syndrome, most likely caused by iatrogenic corticosteroid use for her chronic autoimmune condition.

152
Q

A 29-year-old pregnant woman is diagnosed with Cushing syndrome caused by a pituitary adenoma. She is 20 weeks pregnant and is concerned about the risks to her fetus. Her doctor informs her of the potential complications if the condition is left untreated.

Question: What is the most common fetal complication associated with untreated Cushing syndrome in pregnancy?

A. Fetal macrosomia
B. Preterm delivery
C. Congenital heart defects
D. Neonatal hypocalcemia

A

B. Preterm delivery
Rationale: Untreated Cushing syndrome during pregnancy is associated with an increased risk of preterm delivery and high perinatal morbidity and mortality.

153
Q

A 35-year-old pregnant woman with known Cushing syndrome caused by an adrenal adenoma presents with worsening hypertension and gestational diabetes. She is currently 22 weeks pregnant. Her physician discusses the need for surgery to treat her condition.

Question: What is the definitive treatment for Cushing syndrome in this patient?

A. Laparoscopic adrenalectomy
B. Chemotherapy
C. Methimazole therapy
D. Beta-blockers and calcium supplementation

A

A. Laparoscopic adrenalectomy
Rationale: The definitive treatment for Cushing syndrome caused by an adrenal adenoma is laparoscopic adrenalectomy, which removes the source of excess cortisol production

Management includes surgical resection of adrenal or pituitary adenomas

154
Q

A pregnant woman at 25 weeks gestation is diagnosed with Cushing syndrome caused by bilateral adrenal hyperplasia. She is concerned about the safety of medications and the effects on her fetus.

Question: Which medication used in the management of Cushing syndrome should be used cautiously due to its potential effects on the male fetus?

A. Ketoconazole
B. Antihypertensives
C. Mefisterone
D. Dexamethasone

A

A. Ketoconazole
Rationale: Ketoconazole blocks steroidogenesis but can affect the male fetus by interfering with testicular function, so it must be used with caution in pregnant women.

155
Q

A 30-year-old woman at 26 weeks gestation presents with moon facies, truncal obesity, and high blood pressure. Blood tests confirm Cushing syndrome. Her doctor is concerned about potential maternal complications if the condition is not treated.

Question: What maternal complications are associated with untreated Cushing syndrome during pregnancy?

A. Hypotension and anemia
B. Hypertension, gestational diabetes, and heart failure
C. Deep vein thrombosis and pulmonary embolism
D. Pre-eclampsia and oligohydramnios

A

B. Hypertension, gestational diabetes, and heart failure
Rationale: Untreated Cushing syndrome can lead to serious maternal complications such as hypertension, gestational diabetes, and heart failure, increasing the risk of maternal death.

156
Q

Which of the following is a typical symptom of a prolactinoma?

A. Polyuria
B. Hyperprolactinemia
C. Hypoglycemia
D. Hypercalcemia

A

B. Hyperprolactinemia
Rationale: Prolactinoma causes hyperprolactinemia, leading to symptoms such as amenorrhea and galactorrhea.

157
Q

Which treatment is commonly used for a microadenoma in a pregnant patient with prolactinoma?

A. Chemotherapy
B. Bromocriptine
C. Radiation therapy
D. Surgical resection

A

B. Bromocriptine
Rationale: Bromocriptine, a dopamine agonist, is commonly used to treat microadenomas in prolactinoma patients, including during pregnancy, as it inhibits prolactin production.

158
Q

At what size does a pituitary adenoma become classified as a macroadenoma?

A. 5 mm
B. 8 mm
C. 10 mm
D. 15 mm

A

C. 10 mm
Rationale: A pituitary tumor is classified as a macroadenoma when it is larger than 10 mm in size.

159
Q

What is the preferred surgical approach for macroadenomas?

A. Open cranial surgery
B. Transnasal or transseptal endoscopic resection
C. Radiation therapy
D. Laparoscopic surgery

A

B. Transnasal or transseptal endoscopic resection
Rationale: The preferred surgical treatment for macroadenomas is transnasal or transseptal endoscopic resection, which is minimally invasive.

160
Q

Which of the following is a potential complication of an untreated enlarging microadenoma during pregnancy?

A. Hypertension
B. Visual disturbances
C. Hypotension
D. Polyuria

A

B. Visual disturbances
Rationale: An enlarging microadenoma during pregnancy can cause visual disturbances due to compression of the optic chiasm.

161
Q

A 30-year-old woman presents with amenorrhea, galactorrhea, and mild headaches. She is not pregnant but expresses concern about her fertility. An MRI reveals a pituitary tumor measuring 8 mm in size, and blood tests show elevated prolactin levels.

Question: What is the most likely diagnosis, and what is the initial treatment?

A. Pituitary microadenoma; treat with bromocriptine
B. Pituitary macroadenoma; immediate surgery
C. Adrenal adenoma; treat with corticosteroids
D. Hypothyroidism; treat with levothyroxine

A

A. Pituitary microadenoma; treat with bromocriptine
Rationale: The patient has a microadenoma (8 mm) with hyperprolactinemia. The first-line treatment is bromocriptine, which is a dopamine agonist that inhibits prolactin production.

162
Q

A 32-year-old pregnant woman in her second trimester presents with headaches and visual disturbances. An MRI before pregnancy revealed a 9 mm prolactinoma. Her prolactin levels have increased, and her symptoms are worsening.

Question: What is the best course of action for managing her symptoms during pregnancy?

A. Continue monitoring without intervention
B. Start bromocriptine to reduce tumor size and control symptoms
C. Perform transnasal surgery immediately
D. Wait until after delivery to begin treatment

A

B. Start bromocriptine to reduce tumor size and control symptoms
Rationale: Bromocriptine is the preferred treatment for a microadenoma during pregnancy, as it helps shrink the tumor and controls hyperprolactinemia to relieve symptoms.

163
Q

A 40-year-old woman presents with worsening vision problems and frequent headaches. She was diagnosed with a 12 mm pituitary adenoma last year but declined treatment. She now reports difficulty with peripheral vision.

Question: What is the most appropriate treatment at this stage?

A. Bromocriptine therapy
B. Transnasal endoscopic resection
C. Radiation therapy
D. Observation and follow-up MRI

A

B. Transnasal endoscopic resection
Rationale: The patient has a macroadenoma (>10 mm) with symptoms of visual disturbances due to compression of the optic chiasm. The appropriate treatment is transnasal endoscopic resection to remove the tumor.

164
Q

A pregnant woman at 28 weeks gestation is diagnosed with a 6 mm prolactinoma. She has developed new onset galactorrhea and headaches, but her vision is intact. Her prolactin levels are significantly elevated.
Question: What is the recommended treatment during pregnancy?

A. Wait until after delivery for treatment
B. Start bromocriptine to reduce prolactin levels and control symptoms
C. Immediate transnasal surgery
D. Radiation therapy

A

B. Start bromocriptine to reduce prolactin levels and control symptoms
Rationale: Bromocriptine is a dopamine agonist and is commonly used to manage prolactinomas during pregnancy, especially for microadenomas (≤10 mm). It helps reduce prolactin levels and alleviates symptoms such as galactorrhea and headaches. Surgery is typically reserved for larger tumors (macroadenomas) or cases with significant visual disturbances or rapid tumor growth.

165
Q

A 29-year-old woman presents with amenorrhea and galactorrhea. Her MRI reveals a pituitary tumor measuring 12 mm, and her prolactin levels are significantly elevated. Her physician explains that the tumor is compressing nearby structures, causing headaches and visual field defects.

Question: What is the most appropriate definitive treatment for this patient’s condition?

A. Bromocriptine therapy only
B. Radiation therapy
C. Transnasal or transseptal endoscopic resection
D. Watchful waiting

A

C. Transnasal or transseptal endoscopic resection
Rationale: The patient has a macroadenoma (>10 mm), which is compressing nearby structures and causing visual field defects. The recommended definitive treatment is transnasal or transseptal endoscopic resection, which is a minimally invasive surgical approach to remove the tumor and relieve compression. Bromocriptine can be used for smaller tumors or to control symptoms, but surgery is necessary for larger, symptomatic tumors.

166
Q

What is the most common cause of Sheehan Syndrome?

A. Severe obstetrical blood loss
B. Head trauma
C. Autoimmune thyroiditis
D. Diabetes insipidus

A

A. Severe obstetrical blood loss
Rationale: Sheehan Syndrome occurs due to pituitary ischemia and necrosis following severe obstetrical blood loss during or after childbirth.

167
Q

Which of the following is a hallmark symptom of Sheehan Syndrome?

A. Polyuria
B. Hypotension and failure to lactate
C. Hypertension and galactorrhea
D. Hyperglycemia and weight gain

A

B. Hypotension and failure to lactate
Rationale: Sheehan Syndrome presents with persistent hypotension and failure to lactate postpartum due to a lack of pituitary hormone production.

168
Q

What is the primary treatment for Sheehan Syndrome?

A. Corticosteroid therapy
B. Hormone replacement therapy
C. Beta-blockers
D. Surgery for all patients

A

B. Hormone replacement therapy
Rationale: The main treatment for Sheehan Syndrome is hormone replacement therapy, as the pituitary gland fails to produce essential hormones following ischemia.

169
Q

Which of the following conditions may develop in patients with Sheehan Syndrome due to pituitary damage?

A. Diabetes mellitus
B. Diabetes insipidus
C. Hypoparathyroidism
D. Hyperthyroidism

A

B. Diabetes insipidus
Rationale: Diabetes insipidus can develop in patients with Sheehan Syndrome because of damage to the pituitary gland, leading to a deficiency in antidiuretic hormone (ADH).

170
Q

Which surgical intervention may be required in cases of Sheehan Syndrome with lymphocytic hypophysitis and optic chiasm compression?

A. Transsphenoidal surgery
B. Craniotomy
C. Laparoscopic surgery
D. Radiation therapy

A

A. Transsphenoidal surgery
Rationale: Transsphenoidal surgery is the surgical intervention of choice when there is optic chiasm compression due to lymphocytic hypophysitis that does not respond to corticosteroid therapy.

171
Q

A 35-year-old woman presents with persistent fatigue, low blood pressure, and an inability to lactate after delivering her baby six months ago. She also reports frequent episodes of low blood sugar and occasional dizziness. She experienced severe blood loss during delivery.

Question: What is the most likely diagnosis, and what is the recommended treatment?

A. Addison’s disease; treat with corticosteroids
B. Sheehan Syndrome; treat with hormone replacement therapy
C. Hypothyroidism; treat with levothyroxine
D. Cushing’s syndrome; treat with surgery

A

B. Sheehan Syndrome; treat with hormone replacement therapy
Rationale: The patient’s symptoms, especially persistent hypotension, fatigue, and failure to lactate, along with her history of severe postpartum hemorrhage, point to Sheehan Syndrome. Hormone replacement therapy is the recommended treatment to manage hormone deficiencies.

172
Q

A 30-year-old woman who gave birth five years ago is being evaluated for persistent fatigue, low blood pressure, and hypoglycemia. She reports an inability to lactate after childbirth and has been experiencing frequent urination and excessive thirst. She was diagnosed with postpartum hemorrhage at the time of delivery.

Question: Which additional condition should be considered in this patient due to her symptoms of excessive thirst and frequent urination?

A. Diabetes insipidus
B. Hyperthyroidism
C. Hypoparathyroidism
D. Cushing syndrome

A

A. Diabetes insipidus
Rationale: Diabetes insipidus should be considered in this patient, as she exhibits symptoms of excessive thirst and frequent urination. This condition can occur due to pituitary damage in Sheehan Syndrome, leading to insufficient antidiuretic hormone (ADH) production.

173
Q

A 28-year-old woman presents with persistent hypotension and recurrent hypoglycemia five years after giving birth. She reports that she did not lactate after delivery, and her doctor suspects Sheehan Syndrome.

Question: What is the most likely cause of Sheehan Syndrome in this patient?

A. Pituitary infarction due to severe postpartum hemorrhage
B. Autoimmune destruction of the thyroid
C. Adrenal hyperplasia
D. Hypercalcemia-induced pituitary damage

A

A. Pituitary infarction due to severe postpartum hemorrhage
Rationale: Sheehan Syndrome is caused by pituitary infarction due to severe postpartum hemorrhage. The patient’s symptoms, including hypotension, hypoglycemia, and failure to lactate, are characteristic of this condition.v

174
Q

A 33-year-old woman with Sheehan Syndrome is being treated with hormone replacement therapy. She begins experiencing visual disturbances, and imaging reveals compression of the optic chiasm due to lymphocytic hypophysitis.

Question: What is the next best step in managing this patient?

A. Increase hormone replacement therapy
B. Perform transsphenoidal surgery
C. Begin radiation therapy
D. Start chemotherapy

A

B. Perform transsphenoidal surgery
Rationale: In cases of lymphocytic hypophysitis with optic chiasm compression that is unresponsive to corticosteroids, transsphenoidal surgery is the recommended treatment to relieve pressure on the optic chiasm.

175
Q

A 36-year-old woman is diagnosed with Sheehan Syndrome after experiencing persistent hypotension, failure to lactate, and fatigue for several years following childbirth. She is concerned about managing her long-term health.

Question: What is the mainstay of treatment for this patient to manage her hormone deficiencies?

A. Corticosteroids
B. Hormone replacement therapy
C. Beta-blockers
D. Surgery

A

B. Hormone replacement therapy
Rationale: The mainstay of treatment for Sheehan Syndrome is hormone replacement therapy, which is necessary to replace the deficient hormones produced by the pituitary gland following ischemia and necrosis.